Vous êtes sur la page 1sur 114

GMAT Verbal Study Guide

Table of Contents
Chapter 1 Reading Comprehension ...............................................................................4 Section 1: One Principle .............................................................................................. 5 Section 2: Two Styles .................................................................................................. 8 1. Presentation....................................................................................................... 8 2. Argumentation .................................................................................................. 9 3. Organizational Structure ..................................................................................11 Section 3: Three Subjects........................................................................................... 14 1. Natural Science ............................................................................................... 14 2. Social Science ................................................................................................. 15 3. Business Subject ............................................................................................. 17 Section 4: Four-step Process of Reading ................................................................... 20 1. Analyze the first paragraph............................................................................. 20 2. Skim the passage and get the author's main point........................................... 21 3. Diagram the organization of the passage ........................................................ 23 4. Tackle the questions and correspondently refer to the passage. ..................... 26 Section 5: Five Types of Questions ........................................................................... 29 1. Main Idea Question......................................................................................... 30 2. Recall Question............................................................................................... 35 3. Inference Questions ........................................................................................ 37 4. Critical Reasoning Question ........................................................................... 39 5. Difficult-to-locate Question ............................................................................ 41 Section 6: Six test points............................................................................................ 43 1. Comparison ..................................................................................................... 43 2. Example & Listing.......................................................................................... 43 3. People, Date & Place ...................................................................................... 46 4. Words of Attitude and Transition ................................................................... 47 5. Counter-evidence Indicators ........................................................................... 49 6. Special Punctuation......................................................................................... 51 Chapter 2 Sentence Correction ....................................................................................53 Introduction................................................................................................................ 53
1

Three-step method ..................................................................................................... 54 Section 1: Subject-Verb Agreement .......................................................................... 56 Section 2: Verb Time Sequences ............................................................................... 57 Section 3: Modification.............................................................................................. 58 A. Adjective or adverb as a modifier .................................................................. 58 B. Clause as a modifier ....................................................................................... 58 C. A long phrase as a modifier............................................................................ 59 D. Appositive as a modifier ................................................................................ 60 Section 4: Parallelism ................................................................................................ 61 Section 5: Pronoun..................................................................................................... 62 Section 6: Comparisons ............................................................................................. 63 1. Quality Comparison ........................................................................................ 63 2. Quantity Comparison ...................................................................................... 64 3. Analogy........................................................................................................... 64 Section 7: Choice of Word......................................................................................... 66 Section 8: Idioms ....................................................................................................... 67 Section 9: Sentence Structure .................................................................................... 72 Section 10: Subjunctive Mood................................................................................... 74 Section 11: Ambiguity ............................................................................................... 75 Section 12: Redundancy ............................................................................................ 76 Section 13: Awkward................................................................................................. 77 Section 14: Logicality ................................................................................................ 78 Chapter 3 Critical Reasoning .......................................................................................80 Section 1: Introduction to Critical Reasoning............................................................ 81 1. One Definition: Argument .............................................................................. 81 2. Four elements of an argument......................................................................... 82 3. Seven Common Fallacies................................................................................ 84 4. Three-element Rule......................................................................................... 86 5. Two Traps ....................................................................................................... 87 6. Five Answer Choices ...................................................................................... 88 Section 2: Six Types of Argument............................................................................. 90 1. Deductive Argument ....................................................................................... 90 2. Generalization ................................................................................................. 94 3. Analogy........................................................................................................... 95 4. Causal Reasoning............................................................................................ 96 5. Finding Assumption........................................................................................ 97 6. Business Thinking........................................................................................... 99 Section 3: Eight Types of Question ......................................................................... 100 1. Inference Question ........................................................................................ 100 2. Assumption Question.................................................................................... 102 3. Strengthen Question...................................................................................... 104
2

4. Weaken Question .......................................................................................... 106 5. Paradox Question .......................................................................................... 108 6. Reasoning Question ...................................................................................... 109 7. Complete Question.........................................................................................110 8. Boldface Question..........................................................................................112

If you have any question or suggestion, please email us at contact@microedu.com. Copyright 2004 The Microedu.com. All Rights Reserved. Not for distribution.

Chapter 1 Reading Comprehension


Reading Comprehension on the test day On the test day, you will expect to see three or four reading passages, each followed with three or four questions. The passages presented depend on how well you are performing on the test. However, the questions presented for the same passage do not depend on your performance. In other word, after you are assigned a reading passage, the next question presented for the same passage will not base on your performance on the previous question. Why Reading Comprehension is a nightmare to most students? Most people find the reading compression difficult to prepare because the subject matter is unfamiliar and could be anything. In order to make sure that nobody can take advantages on a particular subject, the test-maker takes every effort to diversify the subjects of the three or four passages on your test day. As a result, obscure subject matter is chosen so that you will be tested, not on your knowledge of a particular subject, but the test-taking skills. In addition, the reading passage is not created like the one we see on magazine, newspaper, or textbook. Rather, it uses a highly compressed style. Subjects of the passages are generally excerpted from academic articles that were published tens of years ago. Usually the chosen article is heavily edited until it is cut down to about 300 words, about one-third its original length. Though it is difficult to read, the reading techniques introduced in the following passage will help you pick up the right answer even without understanding the reading passage. Chapter Preview In order to make it easier for you to prepare for GMAT, we have developed an interesting course for Reading Comprehension. You will find this chapter all in number, as the section number suggests. We hope this would help you learn the test prep strategies. Section 1: One Principle Section 2: Two Writing Styles Section 3: Three Subjects Section 4: Four-step Procedure for Attacking a Passage Section 5: Five Types of Question Section 6: Six Test Points

Section 1: One Principle


Directions: The questions in this group are based on the content of a passage. After reading the passage, choose the best answer to each question. Answer all questions following the passage on the basis of what is stated or implied in the passage On the test day, you will see the above direction on computer screen. Most students disregard this instruction since it appears in every test. However, it introduces a basic principle you should follow in answering a reading comprehension question. When answering questions, you must refer each of them to some place in the passage. Don't rely on memory, since too many traps are used with these questions. Also, don't base on your daily life experiences or college knowledge. Remember, GMAT doesn't test any specific knowledge on business or other functions. Image if a question is based on some specific knowledge, then those with broad knowledge can take advantages. This definitely violates the rule of GMAT. The test-maker often fools you by creating stuff choices that contain reasonable statement based on basic knowledge or your life experience, not on the passage. If you find an answer choice contains the widely known reasoning or statement on the test day, eliminate those choices with hesitation. Let's look at a sample passage that discusses why the Indian software vendors perform better than their counterparts in China. Indian firms have achieved the highest levels of efficiency in the world software outsourcing industry. Some researchers have assumed that Indian firms use the same programming languages and techniques as Chinese firms but have benefited from their familiarity with English, the language used to write software code. However, if this were true, then one would expect software vendors in Hong Kong, where most people speak English, to perform not worse than do Indian vendors. However, this is obviously not the case. Other researchers link high Indian productivity to higher levels of human resource investment per engineer. But a historical perspective leads to a different conclusion. When the two top Indian vendors matched and then doubled Chinese productivity levels in the mid-eighties, human resource investment per employee was comparable to that of Chinese vendors. Furthermore, by the late eighties, the amount of fixed assets required to develop one software package was roughly equivalent in India and in the China. Since human resource investment was not higher in India, it had to be other factors that led to higher productivity. A more fruitful explanation may lie with Indian strategic approach in outsourcing. Indian software vendors did not simply seek outsourced contract more effectively: they made aggressive strategic in outsourcing. For instance, most software firms of India were initially set up to outsource the contract in western countries, such as United States. By contrary,

most Chinese firms seem to position their business in China, a promising yet under-developed market. However, rampant piracy in China took almost 90 percents of potential market, making it impossible for most Chinese firms to obtain sufficient compensation for the investment on development and research, let alone thrive in competitive environment. Now, let's look at a sample question: Which of the following statements concerning the productivity levels of engineers can be inferred from the passage? (A) Prior to the 1980s, the productivity levels of the top Indian software firms were exceeded by those of Chinese software firms. (B) The official language of a country has a large effect on the productivity levels of its software developers. (C) During the late 1980s and early 1990s, productivity levels were comparable in China and India. (D) The greater the number of engineers that a software firm has, the higher a firms productivity level. (E) The amount of human resource investment made by software developers in their firms determines the level of productivity. If you do not refer to the original passage, you may pick up B. For test-takers who have some backgrounds in computer, it is obvious that being familiar with English will gain some advantage in writing program code. However, the correct answer is C. In conclusion, the directions can run out of your eyes on the test day, but should be rooted deeply in your heart at the beginning of your test preparation.

Trap: Some choices just repeat the same words or phrases that you read in the passage. Keep alarm to these choices since in most cases, they are incorrect. Here is an example: The fact that reducing price can generate a competitive advantage for a company does not mean that every reduction in price will create such an advantage. Price reduction, like improvement in service, must be balanced against other types of efforts on the basis of direct, tangible benefits such as increased revenues. If a company is already effectively on a par with its competitors because it provides product at an acceptable price and keeps customers

from leaving at an unacceptable rate, then reduction in price may not be effective, since price is not necessarily the deciding factor for any customer in any situation. This truth was not apparent to managers of one operating system software vendor, which failed to improve its competitive position despite its attempt to reduce price. The software managers did not recognize the level of customer inertia that arises from the inconvenience of switching operating system. Nor did they analyze their reduction in price to determine whether it would attract new customers by producing a new standard of price that would excite customers or by proving difficult for competitors to copy. Sample question According to the passage, reduction in price are comparable to improvement in service in terms of the (A) tangibility of the benefits that they tend to confer (B) increased revenues that they ultimately produce (C) basis on which they need to be weighed (D) insufficient analysis that managers devote to them (E) degree of competitive advantage that they are likely to provide To answer this question, first locate the question to the second sentence of the passage. "Price reduction, like improvement in service, must be balanced against other types of efforts on the basis of direct, tangible benefits such as increased revenues." Now, go back to answer choices. Choice D and E are irrelevant to the original sentence, so eliminate them. Then, look at the choice A and B, both of them repeat the original sentences. (A) (B) (C) tangibility of the benefits that they tend to confer increased revenues that they ultimately produce basis on which they need to be weighed

Both A and B seem to be correct. However, reduction in price is comparable to that of improvement in service in term of the basis on direct and tangible benefits, not on the tangibility or specific benefits of increased revenues. So neither A nor B is correct. Choice B does not repeat the same words, but address the basis for comparison. Therefore, C is the correct answer.

Section 2: Two Styles


There is an endless number of writing techniques that authors use to present their ideas. However, there are only two writing styles used in a GMAT reading passage: presentation and argumentation. 1. Presentation This technique is to present an idea that the author will agree or at least partially agree. The author strengthens his position by citing relevant evidences, each related to other in a highly structured manner. We call this style of writing as presentation. Sometimes, the author sometime may intentionally contrast his position with an opposing view. But most often the author is just anticipating an objection, he will soon refute it. Here is a sample passage in presentation. China as a nation faces two major financial problems. First, eighty-four percent of state-owned enterprises do not generate profit. Government failed to collect money from such business. Rather, it has to appropriate substantial funds to these enterprises in order to prevent them from going bankrupt and thus resulting in high unemployment rate. Second, 203 million of civilians in countryside will not be able to gain pension after they retire due to the limited budget of government. I would like to make an outrageous suggestion that would at one stroke generate finance earnings and provide funds for civilians retirement. I would propose that government sells its holdings in state-owned enterprises on the open market. Such sales would provide substantial funds for village civilians pension. At the same, they could cut down financial burden on these state-owned enterprises. You might object that government would be deprived of the opportunity to share its enterprises profit if someday they make money. I agree. Sell holdings of enterprises that would never generate profit. But, you might reply, every enterprise that competes on the market has potential. Here we part company. Theoretically, you may be correct in claiming that every enterprise has the potential to make money. Practically, you are wrong. I refer to the thousands of state-owned enterprises that are not likely to make money. These companies are 100 percent held by the nation as a whole. Government officials are appointed as the chairman, CEO and president. The management was not responsible for the public interest, but for the nation as a whole. If there is no significant loss in business, they will soon be promoted back to the higher level position in government. If their companies perform great, these executives receive direct money compensation. However,

their salary, when combined with such compensation, will be far below that of their counterpart in private company. It would be unrealistic to suggest that village civilians would have sufficient funds if governments shares were sold on the open market. But the demand for compensating the state-own enterprises would be substantially reduced. The author developed the above passage by first pointing out a problem, suggesting a solution, anticipating counter-position, illustrating an example, refuting a second solution, and further anticipating possible objections. Obviously, this writing technique is presentation. 2. Argumentation The second writing style is argumentation. This technique has a number of variations, but the most common and direct is to develop two to three ideas and then point out why one is better than the other or just simply refute all of them and developed the author's own idea. Some common tip-off sentences to this method of analysis are: It was traditionally assumed... It was once believed... It was frequently assumed .. It was universally accepted.. Many scientists have argued... The passage that discusses Indian and Chinese software firms represents a typical argumentation. At the beginning, the author presented a phenomenon and gave an explanation, but refuted that explanation immediately. Indian firms have achieved the highest levels of efficiency in the world software outsourcing industry. Some researchers have assumed that Indian firms use the same programming languages and techniques as Chinese firms but have benefited from their familiarity with English, the language used to write software code. However, if this were true, then one would expect software vendors in Hong Kong, where most people speak English, to perform not worse than do Indian vendors. However, this is obviously not the case. Then, the second explanation was introduced, but was denied again in the same paragraph. Other researchers link high Indian productivity to higher levels of human resource investment per engineer. But a historical perspective leads to a different conclusion. When the two top Indian vendors matched and then doubled Chinese productivity levels in the mid-eighties, human resource investment per employee was comparable to that of Chinese vendors.
9

Furthermore, by the late eighties, the amount of fixed assets required to develop one software package was roughly equivalent in India and in the China. Since human resource investment was not higher in India, it had to be other factors that led to higher productivity. Finally, a more fruitful one is presented. The author used the remaining passage try to argue that this explanation is the correct one. A more fruitful explanation may lie with Indian strategic approach in outsourcing. Indian software vendors did not simply seek outsourced contract more effectively: they made aggressive strategic in outsourcing. For instance, most software firms of India were initially set up to outsource the contract in western countries, such as United States. By contrary, most Chinese firms seem to position their business in China, a promising yet under-developed market. However, rampant piracy in China took almost 90 percents of potential market, making it impossible for most Chinese firms to obtain sufficient compensation for the investment on development and research, let alone thrive in competitive environment. Why bother to identify the writing style? Be familiar with the author's writing techniques can help you diagram the mental road map of a passage, identify the author's intention to cite an evidence, main idea of a passage, and most importantly, pick up the right choice quickly and decisively. Lets go back the passage that talks about whether price reduction can generate a competitive advantage. The fact that reducing price can generate a competitive advantage for a company does not mean that every reduction in price will create such an advantage. Price reduction, like improvement in service, must be balanced against other types of efforts on the basis of direct, tangible benefits such as increased revenues. If a company is already effectively on a par with its competitors because it provides product at an acceptable price and keeps customers from leaving at an unacceptable rate, then reduction in price may not be effective, since price is not necessarily the deciding factor for any customer in any situation. This truth was not apparent to managers of one operating system software vendor, which failed to improve its competitive position despite its attempt to reduce price. The software managers did not recognize the level of customer inertia that arises from the inconvenience of switching operating system. Nor did they analyze their reduction in price to determine whether it would attract new customers by producing a new standard of price that would excite customers or by proving difficult for competitors to copy. In the above passage, the author did not try to present his own position (presentation). If any, the position is that he does not agree with the fact that reduction in price can generate competitive

10

advantage for a company. In fact, the speaker here argued against a popular point of view by reasoning and examples (argumentation). Let's look at a sample question to see how to pick up a right choice on the basis of writing styles. The primary purpose of the passage is to (A) contrast possible outcomes of a type of business strategy (B) suggest more careful evaluation of a type of business strategy (C) illustrate various ways in which a type of business strategy could fail to enhance revenues (D) trace the general problems of a company to a certain type of business strategy (E) criticize the way in which managers tend to analyze the costs and benefits of business strategies This question asks you to summarize the passage's central idea. Which of the five choices is correct? Based on the verbs initiating the five choices, you can eliminate three of them: (A) incorrect. To contrast is to compare several things, but not to agree or disagree. (C) incorrect. To illustrate is to give example, not to agree or disagree. (D) incorrect. To trace is to track, not to agree or disagree. Choice E began with argumental word criticize, but isn't the correct choice because it addresses the detail. Therefore, B is the right answer: to argue that superior service does not generate competitive advantage is to suggest more careful evaluation of a type of business strategy (price reduction). 3. Organizational Structure There are two major patterns that the test-maker uses to reach a conclusion: general-to-specific and specific-to-general. Become familiar with these writing patterns can help you identify the main idea of a passage. A. General-to-Specific Structure This structure is widely used in GMAT reading passage. The test-writer first makes a general argument, and then supports it using a series of specific examples or reasoning, and finally summaries by reclaiming his general argument. Here is the structure: General claim, followed by first evidence or reasoning second evidence or reasoning
11

more evidence or reasoning Let's look at a passage of this structure: The fact that reducing price can generate a competitive advantage for a company does not mean that every reduction in price will create such an advantage. Price reduction, like improvement in service, must be balanced against other types of efforts on the basis of direct, tangible benefits such as increased revenues. If a company is already effectively on a par with its competitors because it provides product at an acceptable price and keeps customers from leaving at an unacceptable rate, then reduction in price may not be effective, since price is not necessarily the deciding factor for any customer in any situation. This truth was not apparent to managers of one operating system software vendor, which failed to improve its competitive position despite its attempt to reduce price. The software managers did not recognize the level of customer inertia that arises from the inconvenience of switching operating system. Nor did they analyze their reduction in price to determine whether it would attract new customers by producing a new standard of price that would excite customers or by proving difficult for competitors to copy. Here, the author presents his opinion at the beginning of the passage: reduction in price does not necessarily generate a competitive advantage. To support his idea, the author first made reasoning by comparing service improvement and price reduction. Then, in the second paragraph, the author used an example within operating system software industry to further address that reducing price did not improve competitive position. B. Specific-to-General Structure Contrast to the general-to-specific structure, the specific-to-general first presents a group of examples or reasoning and finally draw a conclusion. Here is the structure: first example or reasoning second example or reasoning more example or reasoning Conclusion The passage that discusses Indian software vendors was written in argumentation, and represents a typical passage in specific-to-general structure. Indian firms have achieved the highest levels of efficiency in the world software outsourcing industry. Some researchers have assumed that Indian firms use the same programming languages and techniques as Chinese firms but have benefited from their familiarity with

12

English, the language used to write software code. However, if this were true, then one would expect software vendors in Hong Kong, where most people speak English, to perform not worse than do Indian vendors. However, this is obviously not the case. Other researchers link high Indian productivity to higher levels of human resource investment per engineer. But a historical perspective leads to a different conclusion. When the two top Indian vendors matched and then doubled Chinese productivity levels in the mid-eighties, human resource investment per employee was comparable to that of Chinese vendors. Furthermore, by the late eighties, the amount of fixed assets required to develop one software package was roughly equivalent in India and in the China. Since human resource investment was not higher in India, it had to be other factors that led to higher productivity. A more fruitful explanation may lie with Indian strategic approach in outsourcing. Indian software vendors did not simply seek outsourced contract more effectively: they made aggressive strategic in outsourcing. For instance, most software firms of India were initially set up to outsource the contract in western countries, such as United States. By contrary, most Chinese firms seem to position their business in China, a promising yet under-developed market. However, rampant piracy in China took almost 90 percents of potential market, making it impossible for most Chinese firms to obtain sufficient compensation for the investment on development and research, let alone thrive in competitive environment. In the above passage, the author gave an explanation to a particular event, but refuted it soon, until it came with a convincing one the conclusion.

13

Section 3: Three Subjects


Like writing techniques, GMAT subjects may vary significantly. The author may present how caffeine activates human behavior, discuss what causes Japanese auto companies to perform better than those in USA, or explain the union's effort to organize the employees in public sectors. Various as the subjects may be, there are only three major subjects that a GMAT passage may be discussing about: natural science, social science, and business. As the name indicates, natural science topic includes biology, chemistry, geology, and archeology; social science includes art, literature, and civil rights; business includes marketing, advertising, management, and economics. 1. Natural Science Characteristics Most test-takers find the natural science difficult to read and beyond their knowledge base. If you try to figure out what they are really talking about, in most cases you will fail, because reading passages in this kind of subject are filled of nomenclatures and jargons. The good news, however, is that the sentences are always written in a simple syntax and the questions to be answered after reading passage are typical of the recalled questions. That means, when you successfully locate the "key words", you will find it easy to get the right answer. Strategy Don't memorize the details or try to figure out the author's reasoning. Skim the passage, and get its central idea as well as organizational structure. The following passage is about natural science topic. This passage is a little difficult to understand, but the following questions are much easier to answer. Sample Passage The cutting-edge science is ringing alarm bells. Avian flu virus picked up by pigs can swap genetic materials with another flu virus already in the pig and become a new, hitherto unknown flu virus for which no person, no animal has preexisting immunity. The kind of virus causes a pandemic because it spreads from human to human. If you took a peek into history, it turns out that previous influenza pandemics have similar scenarios. The greatest influenza pandemic in 1918 caused more than 20 million deaths of soldiers stationed in France. The last influenza pandemic was in 1968, known as the Hong Kong flu (H3N2). Thousands of deaths and millions were infected worldwide. The other examples are the Nipah virus and Japanese Encephalitis virus, which find pigs to be good hosts. With JE, the virus circulates in the blood of infected pigs. When infected pigs

14

are bitten by Culex mosquitoes, the virus replicates in the mosquito's gut. The next time the mosquito bites a human, the virus is passed on. The pig doesn't get sick as such. The Nipah virus causes pneumonia symptoms in pigs. In humans, it causes encephalitis, and humans catch it only with direct contact with infected pigs. Symptoms range from mild headache to permanent brain damage, and can be fatal. It's merely a phenomenon of nature that the pig is the "mixing vessel" for the new germ. But make no mistake, the pig is not the villain, neither is the chicken. It's actually us, and our horrible farm practices, outdated agricultural policy and, most of all, reckless disregard of our ecology and environment. "Hygiene and management can control what eventually happens," says Lam. "Good farming practice will prevent serious outbreaks and infection to humans." Despite knowing that, animal diseases and the possibility of transmission to humans are becoming quite alarming. Of the 35 new emerging diseases in the last 20 years, more than 70 per cent involved animals. In fact, what we may have done is unwittingly create the perfect launch pad for an influenza pandemic that will likely kill large numbers of people across the globe. Although scientists say it's impossible to predict the odds that the virus will alter its genetic form radically enough to start leaping from human to human, the longer H5N1 is out there killing chickens, the higher the chances are. Sample Question Which of the following statement can be inferred from the passage? (A) New emerging diseases causes more deaths of human than animal. (B) Animals are the villain for most flues. (C) Hygiene and management can not control the spread of viruses. (D) The current bird flu epidemic may be a launch pad for the next influenza pandemic. (E) The influenza pandemic is always a regional phenomenon. Which answer is correct? For choice A, the passage did not make any comparison between deaths of human and deaths of animal. In B, animal is actually not the villain for most flues. Rather, it is human. Look at the second sentence in the fourth paragraph, But make no mistake, the pig is not the villain, neither is the chicken. For C, Hygiene and management can control what eventually happens(in the middle of fourth paragraph), therefore, C is incorrect. E is also incorrect. Though most flues discussed in this passage were originated from some areas, the passage never stated it was a regional phenomenon. In fact, it will likely kill large numbers of people across the globe, as stated at the beginning of last paragraph. The correct answer is D the current bird flu epidemic may be a launch pad for the next influenza pandemic, because no animal has preexisting immunity and it causes a pandemic by spreading from human to human.
15

2. Social Science Characteristics Why women's rights experienced a significant improvement during 1860's? How the Pullman stroke to improve their living condition? Passages in these subjects are easy to read because it goes as you expect and talks about something around your world. You will find it easy to grasp the main idea and passage map. In order to get the right answer, however, you need to read beyond the words, phrases or concepts in the passage. The right answer is always created in a synthesized way. Strategy Be careful in tackling this "social" passage. To answer the later questions is always not as easy as to understand the passage. The answer choice that contains the exact words or phrases from the passage is generally not the correct answer. Rather, you need to synthesize several sentences or make some reasoning before you pick up the right choice. The process is time-consuming because the social passage is typically long. Sample Passage China as a nation faces two major financial problems. First, eighty-four percent of state-owned enterprises do not generate profit. Government failed to make money from such business. Rather, it has to appropriate substantial funds to these enterprises in order to prevent them from going bankrupt and thus resulting in high unemployment rate. Second, 203 million of civilians in countryside will not be able to gain pension after they retire due to the limited budget of government. I would like to make an outrageous suggestion that would at one stroke generate finance earnings and provide funds for civilians retirement. I would propose that government sells its holdings in state-owned enterprises on the open market. Such sales would provide substantial funds for village civilians pension. At the same time, they could cut down financial burden on these state-owned enterprises. You might object that government would be deprived of the opportunity to share its enterprises profit if someday they make money. I agree. Sell holdings of enterprises that would never generate profit. But, you might reply, every enterprise that competes on the market has potential. Here we part company. Theoretically, you may be correct in claiming that every enterprise has the potential to make money. Practically, you are wrong. I refer to the thousands of state-owned enterprises that are not likely to make money. These companies are 100 percent held by the nation as a whole. Government officials are appointed as the chairman, CEO and president. The management was not responsible for

16

the public interest, but for the nation as a whole. If there is no significant loss in business, they will soon be promoted back to the higher level position in government. If their companies perform great, these executives receive direct money compensation. However, their salary, when combined with such compensation, will be much less than the amount they would earn if were in private company. It would be unrealistic to suggest that village civilians would have sufficient funds if governments shares were sold on the open market. But the demand for compensating the state-own enterprises would be substantially reduced. Sample Question According to the passage, executives in a state-owned enterprise are motivated by (A) direct money compensation (B) increasing salary (C) political outlook (D) share option (E) social responsibility The passage mentioned the executives of state-owned enterprises in fourth paragraph; therefore, we need not to consider other paragraphs when referring to the original passage. Since their salary, when combined with such compensation, will be much less than the amount they would earn if were in private company as stated in the last sentence, these executives are not motivated by financial earnings. If yes, they will transfer to a private company. Therefore, they are not motivated by direct money compensation, increasing salary, or share option. Rather, they are concerned on their political outlook. If there is no significant loss in business, they will soon be promoted back to the higher level position in government. Choice C is the correct answer. For choice E, the passage never discussed the executives social responsibility.

17

3. Business Subject Characteristics This subject is highly welcomed since most students possess some knowledge or background in business. But passage of this subject contains the most difficult questions in GMAT Reading Comprehension. Recall questions are few and you always have to reason before you pick up the correct choice. Strategy Don't rely on your memory even if you become or have been quite familiar with its topics. There are too many traps here. Make sure you refer to the passage when answering the questions. Sample Passage The fact that reducing price can generate a competitive advantage for a company does not mean that every reduction in price will create such an advantage. Price reduction, like improvement in service, must be balanced against other types of efforts on the basis of direct, tangible benefits such as increased revenues. If a company is already effectively on a par with its competitors because it provides product at an acceptable price and keeps customers from leaving at an unacceptable rate, then reduction in price may not be effective, since price is not necessarily the deciding factor for any customer in any situation. This truth was not apparent to managers of one operating system software vendor, which failed to improve its competitive position despite its attempt to reduce price. The software managers did not recognize the level of customer inertia that arises from the inconvenience of switching operating system. Nor did they analyze their reduction in price to determine whether it would attract new customers by producing a new standard of price that would excite customers or by proving difficult for competitors to copy. Sample Question The passage suggests which of the following about price charged by an operating system software vendor prior to its strategy in reducing its price? (A) It was slightly low to that of the vendors competitors. (B) It threatened to weaken the vendors competitive position with respect to other operating system software vendor (C) It had already been reduced after having caused damage to the vendors reputation in the past. (D) It enabled the vendor to retain customers at an acceptable rate

18

(E) It needed to be reduced to attain parity with the software provided by competing vendors. Here, the question was created in complicated clauses and itself already hard to understand. In fact, it asks for the situation of the vendor before price reduction. Only D can be inferred from the passage. The original passage stated that If a company is already effectively on .. keeps customers from leaving at an unacceptable rate and This truth was not apparent to managers of one operating system software vendor That means the vendor was able to retain customers at an acceptable rate.

19

Section 4: Four-step Process of Reading


In the previous section we summarize three kinds of subject you will encounter in a GMAT reading passage. Now you will learn the four-step procedure to read a passage in any subject: 1. 2. 3. 4. Analyze the first paragraph. Skim the passage and get some idea of the main idea Identify the purpose of each paragraph and structure of the passage Answer the questions and don't forget to refer to the passage

1. Analyze the first paragraph. It is essential to carefully read the first paragraph. You will get informed what the passage is talking about, and even the main idea of the passage. There are two major reasons for you to carefully read the first paragraph. Fist of all, the paragraph is the main structural unit of any passage. Every paragraph is needed to understand the whole passage or answer the question after the passage. Test-maker never delivers a junk content. It must talk about something that relates to the central idea, and present it as persuasively as possible. In fact, the first paragraph introduces either the position that the author will support or the one that he/she will argue against. So, getting familiar with the introductory paragraph will definitely help you identify the main topic. Secondly, analyzing the fist paragraph in stead of the whole passage can save you much time. As I said at the beginning of this chapter, GMAT reading passage is dry and unfamiliar. It is highly likely that after you read the passage, you get no ideas about what the passage is talking about. If you go back and reread the whole passage, you will have no sufficient time to answer the question. Analyze the first paragraph, pay attention to concepts, and then you will find it easy to understand the subject of passage. Below is the first paragraph of a GMAT reading passage. Pay attention to concept words. China as a nation faces two major financial problems. First, eighty-four percent of state-owned enterprises do not generate profit. Government failed to make money from such business. Rather, it has to appropriate substantial funds to these enterprises in order to prevent them from going bankrupt and thus resulting in high unemployment rate. Second, 203 million of civilians in countryside will not be able to gain pension after they retire due to the limited budget of government. The first sentence stated that China faces two problems. Then, the author specified these two problems using a clear and logical structure. Firstly, government did not make money from but

20

input large amount of money to its enterprises. Secondly, government has limited funds for pension. Now, let's summarize this paragraph and put it in our own words-- China has two problems: financial burden and limited funds. Keep these key words (concepts) in mind, and you will find it easy to understand the remaining passage that we'll present in next step. 2. Skim the passage and get the author's main point Here are some strategies that will speed your reading and help you identify the author's main points: Focus on the first sentence of each paragraph The first sentence of a paragraph is always the main point of this paragraph. Why? It confirms to the formal writing style. If you are a management consultant, you will find it a great advantage to use a summary at the very beginning of each section. Image when you are presenting a strategy report which contains hundreds of pages, how could your clients catch your all of them? The only solution is to use a highly structured presentation, and summarize your idea at the beginning of each section. In fact, you are also doing like this in the AWA section. By simply reading the first sentence of each paragraph, you can construct a mental road map of the passage while not spending significant time. Pay attention to the mood words "Mood" words are those that the author uses to demonstrate his/her position to a particular event, phenomenon, or point of view. A mood word can be positive or negative. Positive words such as successfully, correctly and right often illustrate an idea that the author agree. And vis-a-vis, a negative word indicates an idea that will be weakened in later passage. The following sentences express the author's position by using positive mood words: a) Haney's through research provides convincing field evidence that.. b) For many yeas, Benjamin Quarles' seminal account of the participation of African Americans in the American Revolution has remained the standard work in the field. c) Roger Rosenblatt's book successfully alters the approach taken by most previous studies. By contrast, the following mood words are negative. fail misunderstand sound ignore misrepresent convincingly overestimate overlook successfully underestimate exaggerate correctly

21

Never ignore the counter-evidence indicators The author uses counter-evidence words not to argue against himself, but concede certain minor points that may weaken his argument. The counter evidence is finally refuted by further evidence. You should keep alarm to these words since some students often mistake them as introducing arguing against a statement. Following are some of the most common used counter-evidence indicators: actually even though however despite nonetheless In spite of admittedly nevertheless do except although may

OK. Let's go back to the passage talking about national finance. Here are the other five paragraphs. In order for you to skim the passage using the above three techniques, we underlined the first sentences of each paragraph, boldfaced the mood words and italicize the counter-evidence indicators. I would like to make an outrageous suggestion that would at one stroke generate finance earnings and provide funds for civilians retirement. I would propose that government sells its holdings in state-owned enterprises on the open market. Such sales would provide substantial funds for village civilians pension. At the same time, they could cut down financial burden on these state-owned enterprises. You might object that government would be deprived of the opportunity to share its enterprises profit if someday they make money. I agree. Sell holdings of enterprises that would never generate profit. But, you might reply, every enterprise that competes on the market has potential. Here we part company. Theoretically, you may be correct in claiming that every enterprise has the potential to make money. Practically, you are wrong. I refer to the thousands of state-owned enterprises that are not likely to make money. These companies are 100 percent held by the nation as a whole. Government officials are appointed as the chairman, CEO and president. The management was not responsible for the public interest, but for the nation as a whole. If there is no significant loss in business, they will soon be promoted back to the higher level position in government. If their companies perform great, these executives receive direct money compensation. However, their salary, when combined with such compensation, will be much less than the amount they would earn if were in private company.

22

It would be unrealistic to suggest that village civilians would have sufficient funds if governments shares were sold on the open market. But the demand for compensating the state-own enterprises would be substantially reduced. What is the main idea of the passage? In a word, the author is to present a solution to funding civilians pension while benefiting the state-owned enterprises. 3. Diagram the organization of the passage You got main idea of each paragraph. Now, its time to ask yourself why the author includes them, what the purpose of each paragraph is, and how each paragraph relates to other. This will help you diagram the organization of a passage, and locate the details when you answer the questions. Pivotal words can help you in diagramming the organization. Pivotal words are signal words or phrases that would in advance indicate the idea of paragraphs. Below represents the most frequently used pivotal words or sentences you will see in a reading passage. Note: A and B represent something, while sb represents somebody. Introduction When it comes to ..., some think ... There is a public debate today that ... A is a common way of ..., but is it a wise one? Recently the problem has been brought into focus. Presenting Opinion Now there is a growing awareness that... It is time we explore the truth of ... Nowhere in history has the issue been more visible. Further Presenting Opinion ... but that is only part of the history. Another equally important aspect is ... A is but one of the many effects. Another is ... Besides, other reasons are... Anticipating Objections You may reply that.

23

Admittedly, .. It is reasonable to expect... It is not surprising that... Exampling For example(instance),... ... such as A,B,C and so on (so forth) A good case in point is... A particular example for this is... Presenting Reasons There are many reasons for ... Why .... , for one thing,... The answer to this problem involves many factors. Any discussion about this problem would inevitably involves ... The first reason can be obliviously seen. Most people would agree that... Some people may neglect that in fact ... Others suggest that... Part of the explanation is ... Comparing The advantages for A for outweigh the disadvantages of... Although A enjoys a distinct advantage ... Indeed , A carries much weight than B when sth is concerned. A maybe ... , but it suffers from the disadvantage that... Transitioning To understand the truth of ..., it is also important to see... A study of ... will make this point clear Further Anticipating Objections Certainly, B has its own advantages, such as... I do not deny that A has its own merits.
24

Conclusion From what has been discussed above, we may safely draw the conclusion that ... In summary, it is wiser ... In short... In step 2, you are assigned to skim the passage and get the main idea. Here, let's identify the purpose of each paragraph for the archeology passage to better understand the passage. (First of all, the author presented the problems) China as a nation faces two major financial problems. First, eighty-four percent of state-owned enterprises do not generate profit. Government failed to make money from such business. Rather, it has to appropriate substantial funds to these enterprises in order to prevent them from going bankrupt and thus resulting in high unemployment rate. Second, 203 million of civilians in countryside will not be able to gain pension after they retire due to the limited budget of government. (Then, the author suggested a solution to the problems) I would like to make an outrageous suggestion that would at one stroke generate finance earnings and provide funds for civilians retirement. I would propose that government sells its holdings in state-owned enterprises on the open market. Such sales would provide substantial funds for village civilians pension. At the same time, they could cut down financial burden on these state-owned enterprises. (Here, the author anticipated a possible objection) You might object that government would be deprived of the opportunity to share its enterprises profit if someday they make money. I agree. Sell holdings of enterprises that would never generate profit. But, you might reply, every enterprise that competes on the market has potential. Here we part company. Theoretically, you may be correct in claiming that every enterprise has the potential to make money. Practically, you are wrong. (Then, the author gave an example to deny this objection) I refer to the thousands of state-owned enterprises that are not likely to make money. These companies are 100 percent held by the nation as a whole. Government officials are appointed as the chairman, CEO and president. The management was not responsible for the public interest, but for the nation as a whole. If there is no significant loss in business, they will soon be promoted back to the higher level position in government. If their companies perform great, these executives receive direct money compensation. However,

25

their salary, when combined with such compensation, will be much less than the amount they would earn if were in private company. (Finally, the author further anticipated a possible objection) It would be unrealistic to suggest that village civilians would have sufficient funds if governments shares were sold on the open market. But the demand for compensating the state-own enterprises would be substantially reduced. Now, you are able to create a mental road map for the whole passage: Paragraph # 1: introduced two major problems that China faces. Paragraph # 2: suggested a solution and explained why it is effective. Paragraph # 3: anticipated a possible objection and denied it soon. Paragraph # 4: exemplified to argue against a position initiated in the third paragraph. Paragraph # 5: concluded that his solution is not perfect, but really effective By making such a road map, I bet you understand this passage quite well. 4. Tackle the questions and correspondently refer to the passage. Now that you have grasped main idea and the organizational structure of the passage, you are about to answer the following questions. Again, dont base on your memory. Always refer to the original passage before you pick up a choice. China as a nation faces two major financial problems. First, eighty-four percent of state-owned enterprises do not generate profit. Government failed to make money from such business. Rather, it has to appropriate substantial funds to these enterprises in order to prevent them from going bankrupt and thus resulting in high unemployment rate. Second, 203 million of civilians in countryside will not be able to gain pension after they retire due to the limited budget of government. I would like to make an outrageous suggestion that would at one stroke generate finance earnings and provide funds for civilians retirement. I would propose that government sells its holdings in state-owned enterprises on the open market. Such sales would provide substantial funds for village civilians pension. At the same time, they could cut down financial burden on these state-owned enterprises. You might object that government would be deprived of the opportunity to share its enterprises profit if someday they make money. I agree. Sell holdings of enterprises that would never generate profit. But, you might reply, every enterprise that competes on the

26

market has potential. Here we part company. Theoretically, you may be correct in claiming that every enterprise has the potential to make money. Practically, you are wrong. I refer to the thousands of state-owned enterprises that are not likely to make money. These companies are 100 percent held by the nation as a whole. Government officials are appointed as the chairman, CEO and president. The management was not responsible for the public interest, but for the nation as a whole. If there is no significant loss in business, they will soon be promoted back to the higher level position in government. If their companies perform great, these executives receive direct money compensation. However, their salary, when combined with such compensation, will be much less than the amount they would earn if were in private company. It would be unrealistic to suggest that village civilians would have sufficient funds if governments shares were sold on the open market. But the demand for compensating the state-own enterprises would be substantially reduced. 1. The primary purpose of the passage is to propose (A) an alternative to manage government property (B) a way to relieve government burden while providing funds to village civilians (C) a way to distinguish state-owned enterprises that make money from those that do no make money (D) the governmental approach to evaluate state-owned enterprise executives (E) a new system for national pension system This question requires you to identify the primary concern of the passage as a whole. The first paragraph introduces two major problems that China faces. The second paragraph suggests a solution and explains why it is effective. The third anticipates a possible objection and refutes it soon. The fourth paragraph illustrates an example to support the authors argument. In the last paragraph, the author concludes that his solution is not perfect, but really effective. Therefore, the correct answer is B. 2. The author implies that all of the following statements about enterprises with which government holds 100 percent share are true EXCEPT: (A) A market for governments share already exists. (B) Such enterprises seldom generate profit. (C) There is likely to be a continuing loss of such enterprises. (D) Government officers are appointed as the executives with such enterprises.

27

(E) If the executives perform poorly, they will be demoted to lower position. The question requires you to identify the answer choice that CANNOT be inferred from the passage. Nothing in the passage implies that the executives will be demoted to lower position if they perform poorly. Therefore, the best answer is E. In answering the question that contains EXCEPT, keep alarm not to be fooled by the test maker. 3. The author implies that which of the following would occur if governments shares were sold on the open market? I. The shortage of retirement fund in village would eventually cease completely. II. Current executives in state-owned enterprises are not motivated to perform better III. Civilians in countryside would be able to seek sufficient funds from government. (A) I only (B) II only (C) I and II only (D) II and III only (E) I, II, and III This question asks you to identify information that is suggested rather than directly stated in the passage. To answer it, first look for the location in the passage of the information specified in answer choice. The last paragraph states that It would be unrealistic to suggest that village civilians would have sufficient funds if governments shares were sold on the open market, therefore, I is incorrect. III, which is a repeated of I, is also incorrect. Only II can be inferred from the original passage, therefore B is the best answer.

28

Section 5: Five Types of Questions


While the techniques introduced in previous four sections speed your reading, this section is developed to help you pick up the right choice quickly and decisively. In the following passage, we will discuss the major question types you may encounter in real GMAT test. Generally, there are only five major types of questions. As you become familiar with the following question types, you will gain an intuitive sense for the places from which questions are likely to be drawn. Note, the order in which the questions are asked roughly corresponds to the order in which the main issues are presented in the passage. Early questions should correspond to information given early in the passage, and so on. Of course, there are many other kinds of classification according to different criteria. Here, we classify, by how we solve reading comprehension questions, into five based on the summary of thousands of the previous real questions. Let's preview the five question types. Question Types Preview 1. Main Idea Question a) b) c) d) 2. a) b) 3. 4. Main Topic Tone Structure Exemplifying Description Listing

Recall Question

Inference Question Critical Reasoning Question a) b) Analogy Assumption/Weaken/Strengthen

5.

Unable-to-locate Question

29

1. Main Idea Question There are four sub-types for this kind of question: Main Topic, Tone, Structure, and Exemplifying. Why should we incorporate them into one type of question? In answering Main Idea Question, you should understand the organizational structure of the passage, the author tone toward a particular point of view in the reading passage, the purpose of each paragraph and why a particular example was illustrated. In other words, if you can determine the main topic of the passage, you are simultaneously well informed with the structure, the intent of specific example, and tone toward specific position. A. Main Topic Main idea questions test your ability to identify and understand an author's intent in a passage. The main idea is usually stated in the first or last paragraph. Main idea questions are usually the first questions asked. Some common main idea questions include: The primary purpose of the passage is to Which of the following titles would best describe the content of the passage? The passage supplies information that would answer which of the following questions? Which of the following is the principal topic of the passage? The passage is most probably an excerpt from. Which of the following best states the central idea of the passage? In most cases, main idea questions are easy to solve. In most GMAT passage the author's primary purpose is to persuade the reader to accept her opinion. Occasionally, it is to describe something. By determining the relationship of each paragraph, you come up with the main ideal at the same time. However, the GMAT writers may obscure the correct answer by surrounding it with close answer choices that stress specifics. Eliminate these choices without hesitation on the test day. Trap 1: The main topic will not focus on certain details in the passage. If you encounter the main ideal question, eliminate the answer choices that describe the details. Trap 2: Pay special attention to the "repeat" answer. Certain choices may exactly repeat some or most words of the correct answer, but do not present the central idea, therefore, is not the correct answer. B. Tone Question Tone questions ask you to identify the writer's attitude. Is the writer's feeling toward the subject positive, negative, or neutral? The following represents some ways of the questions asked.

30

Which of the following best summarizes the author's evaluation of Bailyn's fourth proposition? The author's attitude toward the culture in most factories is best described as (A) cautious (B) critical (C) disinterested (D) respectful (E) adulatory

However, if you did not get a feel for the writer's attitude on the first reading, check the mood words that he chooses. Beware of answer choices that contain extreme emotions. Remember the passages are taken from academic journals. In the rarefied air of academic circles, strong emotions are considered inappropriate and sophomoric. The writers want to display opinions that are considered and reasonable, not spontaneous and off-the-wall. So if an author's tone is negative, it may be disapproving, not snide or ridiculous. If her tone is positive, it may be approving, not ecstatic. Or if her tone is neutral, it would be not be disinterested. C. Organizational Structure When you can determine the right answer for main topic, generally you have been familiar with the organizational structure. Every passage is consisted of some paragraphs, and each single paragraph performs some certain function to the passage as a whole, by presenting, supporting or refuting the central idea. So, think about the purpose of each paragraph as you read through the passage. You may encounter one type of question concerning the main idea or purpose of some certain paragraph. Some common questions include: The last paragraph of the passage performs which of the following functions? Which of the following best describes the organization of the second paragraph? Which of the following best describes the organization of the passage? Which of the following best describes the relation of the first paragraph to the passage as a whole? It is relatively easy to solve this kind of question for two reasons. First, however the question may ask, it is concerning the main idea of the paragraph. If you come up with a question including "paragraph", it definitely require you to generate that paragraph. Second, paragraph is only some element of the completed passage. If you can generate the main topic for the whole passage of three to four paragraphs, why aren't you able to summarize just one paragraph? D. Exemplifying The other type of structure question, exemplifying, tests your ability to identify the intention of author's illustrating of something, some people, or phenomenon. In answering this question, you need to first locate the example, and then refer to opinion preceding or accompanying the example. The right answer is the repeat of this opinion. In illustrating the example in line 13-16, the author intended to.?

31

The author referred to the experiment in order to.? In addition to the above four types, the Main Idea Question may require you to respond in other ways, such as: From what kinds of subject could this passage be excerpted? Which of the following topic would be preceding this passage? This question, however, is similar to the above four, since they are based on your understanding of the completed passage. Sample Question #1 The cutting-edge science is ringing alarm bells. Avian flu virus picked up by pigs can swap genetic materials with another flu virus already in the pig and become a new, hitherto unknown flu virus for which no person, no animal has preexisting immunity. The kind of virus causes a pandemic because it spreads from human to human. If you took a peek into history, it turns out that previous influenza pandemics have similar scenarios. The greatest influenza pandemic in 1918 caused more than 20 million deaths of soldiers stationed in France. The last influenza pandemic was in 1968, known as the Hong Kong flu (H3N2). Thousands of deaths and millions were infected worldwide. The other examples are the Nipah virus and Japanese Encephalitis virus, which find pigs to be good hosts. With JE, the virus circulates in the blood of infected pigs. When infected pigs are bitten by Culex mosquitoes, the virus replicates in the mosquito's gut. The next time the mosquito bites a human, the virus is passed on. The pig doesn't get sick as such. The Nipah virus causes pneumonia symptoms in pigs. In humans, it causes encephalitis, and humans catch it only with direct contact with infected pigs. Symptoms range from mild headache to permanent brain damage, and can be fatal. It's merely a phenomenon of nature that the pig is the "mixing vessel" for the new germ. But make no mistake, the pig is not the villain, neither is the chicken. It's actually us, and our horrible farm practices, outdated agricultural policy and, most of all, reckless disregard of our ecology and environment. "Hygiene and management can control what eventually happens," says Lam. "Good farming practice will prevent serious outbreaks and infection to humans." Despite knowing that, animal diseases and the possibility of transmission to humans are becoming quite alarming. Of the 35 new emerging diseases in the last 20 years, more than 70 per cent involved animals. In fact, what we may have done is unwittingly create the perfect launch pad for an influenza pandemic that will likely kill large numbers of people across the globe. Although scientists say it's impossible to predict the odds that the virus will alter its genetic form radically enough
32

to start leaping from human to human, the longer H5N1 is out there killing chickens, the higher the chances are. Which of the following best describes the topic of the passage? (A) What causes the Nipah virus and Japanese Encephalitis virus to happen? (B) Does Hong Kong flu originate from pig? (C) From fowl to pigs to humans? (D) Is influenza pandemic horrible? (E) Shall we eat chicken? This question asks you to find a title for the passage. In other word, it requires you to identify the primary concern of the passage as a whole. The first paragraph presents a recent virus. The second and third paragraphs describe similar influenza pandemics in history. The fourth paragraph concludes who should be responsible for the spread of virus and what human can do to control. The last paragraph indicates that people stimulated rather than inhibited its promulgation. We can thus conclude the current virus will also leap to human. Furthermore, the passage as a whole is to ring alarm bells. Therefore, C is the best answer. Sample Question #2 Indian firms have achieved the highest levels of efficiency in the world software outsourcing industry. Some researchers have assumed that Indian firms use the same programming languages and techniques as Chinese firms but have benefited from their familiarity with English, the language used to write software code. However, if this were true, then one would expect software vendors in Hong Kong, where most people speak English, to perform not worse than do Indian vendors. However, this is obviously not the case. Other researchers link high Indian productivity to higher levels of human resource investment per engineer. But a historical perspective leads to a different conclusion. When the two top Indian vendors matched and then doubled Chinese productivity levels in the mid-eighties, human resource investment per employee was comparable to that of Chinese vendors. Furthermore, by the late eighties, the amount of fixed assets required to develop one software package was roughly equivalent in India and in the China. Since human resource investment was not higher in India, it had to be other factors that led to higher productivity. A more fruitful explanation may lie with Indian strategic approach in outsourcing. Indian software vendors did not simply seek outsourced contract more effectively: they made aggressive strategic in outsourcing. For instance, most software firms of India were initially set up to outsource the contract in western countries, such as United States. By contrary, most Chinese firms seem to position their business in China, a promising yet

33

under-developed market. However, rampant piracy in China took almost 90 percents of potential market, making it impossible for most Chinese firms to obtain sufficient compensation for the investment on development and research, let alone thrive in competitive environment. Which of the following best describes the organization of the first paragraph? (A) A thesis is presented and supporting examples are provided. (B) Opposing views are presented, classified, and then reconciled. (C) A fact is stated, and an explanation is advanced and then refuted. (D) A theory is proposed, considered, and then amended. (E) An opinion is presented, qualified, and then reaffirmed. This question requires you to identify the organizational structure of the first paragraph. In this paragraph, the author first states a fact that Indian firms achieved the highest efficiency in software outsourcing. Then, an assumption is presented to explain such phenomenon. However, the author refuted this explanation soon. Thus, C is the best answer.

34

2. Recall Question There are two subtypes of recall questions: detail-locating and listing. In the following passage, we'll discuss one by one. A. Detail-locating Locating question is the most common question you will encounter in Reading Comprehension. It roughly constitutes to 50-60% of total numbers of questions. That means, in every reading passage, there will be about one or two detail-locating questions. It is quite simple, however, to solve this seemingly difficult question if you are able locate the detail tested. The right answer choice is rewritten from certain sentence in the passage by changing some words or phrases. For example, test writer will change some words from adjective to adverbial, from noun to gerund, or just change to its synonym. Strategy: How to locate Below we will introduce the three-step method to locate detail. (1) Before you locate the question to passage, you need to determine what to locate. Key words are something that is mentioned both in the question and in passage. Then, what are key words? Look at the following question: Which of the following is mentioned in the passage as a disadvantage of storing artifacts in museum basements? Here, key words will not be any word or phrase of "which of the following is mentioned in the passage as", but will be "disadvantage" from "disadvantage of storing artifacts in museum basements". We call this step as Defining Key Words. (2) After you define key words, you are turning to the original passage. Sometimes, the key words will appear several times in different parts of the passage. Where should you refer to? Generally, you should locate the key words to the sentence in which key words first appear. After all, you have only several minutes to complete a passage. (3) When you determined which sentence (sometimes, two or more sentences) to locate, get some idea, then quickly refer to the answer choices. Do not spend too much time analyzing this sentence since it may be too long or complicated to understand. If the choice mentions something that only appears in other part of the passage, eliminate it. Also eliminate the choice that just repeats the words or phrases from original passage. B. Listing The other type of Recall Question is listing. As the name indicates, Listing Question requires you to identify some people, actions, or situations that are enumerated in the passage.
35

Here are some Listing Questions: According to the passage, senior managers use intuition in all of the following ways EXCEPT to: According to the passage, critics of the Ewha women's studies program cited the program as a threat to which of the following? I. II. National identity National unification

III. Economic development IV. Family integrity (A) I only (B) I and II only (C) I, II, and III only (D) II, III, and IV only (E) I, II, III, and IV To solve this type of question, you should first name the key word from the stimulus, and locate it to original passage. Then, you will find some lists that are similar to the answer choices. Carefully compare those lists one by one to the answer choices. Use POE to eliminate incorrect choice, until you find the right one.

36

3. Inference Questions Inference question is the second most common. Unlike recall question, inference questions require you to go beyond the passage. That means, the correct answer must say more than what is said in the passage. Beware of same language traps with these questions: the correct answer will often both paraphrase and extend a statement in the passage, but it will not directly quote it. If you are puzzled how to determine whether a detail question is recall question or inference question, pay attentions to the way the question asks. Generally, inference question will include some word, such as infer, suggest and imply that indicates what kind of question it is. It can be inferred from the passage that... The passage/author suggests that. The passage/author implies that... Since we must not directly refer to the original passage in answering inference question, we need to decipher the inference. Next, we will show you how to reason from couples of sentence. Technique 1 Reasoning by Word of Comparison The question is asking about B, but you may be unable to directly identify the characters of B even you have located B. Rather, the original sentence is discussing about A. Here, you should turn to the word that indicates comparison between A and B. Some words that indicate strong comparison are unlike, in contrast to, by contrast and compared with. When you can determine the character of B, you can simultaneously determine A is B or non-B. Also, the passage may compare two particular events by dates or places. The phrases could be "prior to 1975" or "since mid-1970's". Technique 2 Reasoning by Syllogism In logics, Syllogism looks like this: every virtue is laudable; kindness is a virtue; therefore, kindness is laudable. As we put it in more simple way, it may be "AB and, then AC". It may be relatively easy to recognize AB by locating the key word in the question, but it will always take some time to identify BC, since they may be located in other part of the place. So pay attention to the pronouns (it or they) and the nouns with definite article "the" since they often serve as B. The fact that reducing price can generate a competitive advantage for a company does not mean that every reduction in price will create such an advantage. Price reduction, like improvement in service, must be balanced against other types of efforts on the basis of direct, tangible benefits such as increased revenues. If a company is already effectively on a par with its competitors because it provides product at an acceptable price and keeps customers from leaving at an unacceptable rate, then reduction in price may not be effective, since price is not necessarily the deciding factor for any customer in any situation.

37

This truth was not apparent to managers of one operating system software vendor, which failed to improve its competitive position despite its attempt to reduce price. The software managers did not recognize the level of customer inertia that arises from the inconvenience of switching operating system. Nor did they analyze their reduction in price to determine whether it would attract new customers by producing a new standard of price that would excite customers or by proving difficult for competitors to copy. The passage suggests which of the following about price charged by an operating system software vendor prior to its strategy in reducing its price? (A) It enabled the vendor to retain customers at an acceptable rate (B) It threatened to weaken the vendors competitive position with respect to other operating system software vendor (C) It had already been reduced after having caused damage to the vendors reputation in the past. (D) It was slightly low to that of the vendors competitors. (E) It needed to be reduced to attain parity with the software provided by competing vendors. Here, the question asks for the situation of the vendor before price reduction. Only A can be inferred from the passage. The original passage stated that If a company is already effectively on .. keeps customers from leaving at an unacceptable rate and This truth was not apparent to managers of one operating system software vendor That means the vendor was able to retain customers at an acceptable rate.

38

4. Critical Reasoning Question Even in reading comprehension, you will encounter some critical reasoning questions: analogy, assumption, weaken, and strengthen. Here, the whole passage is an argument with premises, assumptions and conclusions. The question asks you to identify the reasoning, critique the argument or recognize the potential assumption. When you need to do is also to first locate the conclusion to particular sentence of the passage, then identify the evidence and conclusion. The premise (or evidence) could be near to the conclusion, or in other part of the 3-4 paragraph passage. So, it is more difficult than its counterpart in critical reasoning section. That's why it appears more often in high difficult level screen. A. Analogy Also known as application question, analogy question requires you to identify the author's reasoning somewhere in the passage and then ask you to select one from the following five answer choices that reasons as that is presented in the passage. The answer choices are generally long and complicated, but they are not so difficult to understand. After you locate the details to certain sentences in the passage, try to identify the reasoning, and then turn to the answer choices. Fortunately, once you identify the reasoning, you will quickly get the right answer since there are great differences among these five choices. B. Assumption/Weaken/Strengthen Weaken, Support, and Assumption are the other three types of question you are expected to solve in critical reasoning question. As we said above, you need to evaluate the argument and identify the assumptions. Typical questions would be: Which of the following, if true, would most weaken the theory proposed by Snyder et al? Which of the following, if true, would most strongly support Keyssar's findings as they are described by the author? Sample Question The cutting-edge science is ringing alarm bells. Avian flu virus picked up by pigs can swap genetic materials with another flu virus already in the pig and become a new, hitherto unknown flu virus for which no person, no animal has preexisting immunity. The kind of virus causes a pandemic because it spreads from human to human. If you took a peek into history, it turns out that previous influenza pandemics have similar scenarios. The greatest influenza pandemic in 1918 caused more than 20 million deaths of soldiers stationed in France. The last influenza pandemic was in 1968, known as the Hong Kong flu (H3N2). Thousands of deaths and millions were infected worldwide.
39

The other examples are the Nipah virus and Japanese Encephalitis virus, which find pigs to be good hosts. With JE, the virus circulates in the blood of infected pigs. When infected pigs are bitten by Culex mosquitoes, the virus replicates in the mosquito's gut. The next time the mosquito bites a human, the virus is passed on. The pig doesn't get sick as such. The Nipah virus causes pneumonia symptoms in pigs. In humans, it causes encephalitis, and humans catch it only with direct contact with infected pigs. Symptoms range from mild headache to permanent brain damage, and can be fatal. It's merely a phenomenon of nature that the pig is the "mixing vessel" for the new germ. But make no mistake, the pig is not the villain, neither is the chicken. It's actually us, and our horrible farm practices, outdated agricultural policy and, most of all, reckless disregard of our ecology and environment. "Hygiene and management can control what eventually happens," says Lam. "Good farming practice will prevent serious outbreaks and infection to humans." Despite knowing that, animal diseases and the possibility of transmission to humans are becoming quite alarming. Of the 35 new emerging diseases in the last 20 years, more than 70 per cent involved animals. In fact, what we may have done is unwittingly create the perfect launch pad for an influenza pandemic that will likely kill large numbers of people across the globe. Although scientists say it's impossible to predict the odds that the virus will alter its genetic form radically enough to start leaping from human to human, the longer H5N1 is out there killing chickens, the higher the chances are. All of the following situations are similar to the spread of avian flu virus described in the first paragraph EXCEPT: (A) The BT2 spread from a pig to another pig, and thus causes significant disease in pig. (B) The AIDS viruses transferred from monkeys to man and spread across the world. (C) The SARS virus originates from some wildlife and is picked up by civet cats from which humans got it. (D) Nipah virus circulates in the blood of infected pig, which is bitten by Culex mosquitoes, the virus replicates in the mosquito's gut. The next time the mosquito bites a human, the virus is passed on. (E) H5N1 starts in chickens and leaps from human to human. The question requires you to recognize a situation that is not similar to the spear of avian flu. Before considering following answer choices, we fist define its rationale. It is something like this: Avian flu virus picked up by pigs and is transferred to human. All of the situations described in the answer choices are similar to it ex that in choice A (from animal to animal). Therefore, A is the best answer.

40

5. Difficult-to-locate Question Some question does not ask for the central idea of a passage. Rather, it requires you to draw a conclusion based on the passage: According to the passage, which of the following is the author most likely to agree with? The passage supplies information that would answer which of the following questions? Unlike Recall Question or Inference Question, Difficult-to-locate Question does not contain key words that you can use to locate the details tested. In order to solve this type of question, you have to skim through the passage again and again until you get the right answer. Eliminating wrong choices often take considerable time since the answer choices are often too long and complicated to understand. That is why most test takers regard difficult-to-locate question as the most difficult one in reading comprehension. The good news is that if you encounter several questions like these, then you probably get a high score since questions are presented based on your performance on the previous questions. Sample Question Indian firms have achieved the highest levels of efficiency in the world software outsourcing industry. Some researchers have assumed that Indian firms use the same programming languages and techniques as Chinese firms but have benefited from their familiarity with English, the language used to write software code. However, if this were true, then one would expect software vendors in Hong Kong, where most people speak English, to perform not worse than do Indian vendors. However, this is obviously not the case. Other researchers link high Indian productivity to higher levels of human resource investment per engineer. But a historical perspective leads to a different conclusion. When the two top Indian vendors matched and then doubled Chinese productivity levels in the mid-eighties, human resource investment per employee was comparable to that of Chinese vendors. Furthermore, by the late eighties, the amount of fixed assets required to develop one software package was roughly equivalent in India and in the China. Since human resource investment was not higher in India, it had to be other factors that led to higher productivity. A more fruitful explanation may lie with Indian strategic approach in outsourcing. Indian software vendors did not simply seek outsourced contract more effectively: they made aggressive strategic in outsourcing. For instance, most software firms of India were initially set up to outsource the contract in western countries, such as United States. By contrary, most Chinese firms seem to position their business in China, a promising yet under-developed market. However, rampant piracy in China took almost 90 percents of potential market, making it impossible for most Chinese firms to obtain sufficient
41

compensation for the investment on development and research, let alone thrive in competitive environment. According to the passage, which of the following statements is true of Indian software developers? (A) Their productivity levels did not equal those of Chinese software engineers until the late eighties. (B) Their high efficiency levels are a direct result of English language familiarity. (C) They develop component-specific software. (D) They are built to outsource the western orders. (E) They develop more packages of software than do those in Chinese developers. In the middle of the last paragraph, the author states that For instance, most software firms of India were initially set up to outsource the contract in western countries, such as United States. Thus, the best answer is D.

42

Section 6: Six test points


While four-step procedure helps you to understand a passage and the five types of question guide you how ETS test the understanding of the passage, the six test points will in advance introduce what would be tested even before you read the questions. As you are reading the passage, keep alarm to certain words or phrases since they would later act as clues for answering the following questions. We call these signal words or phrases as test points. In the following passage, we will introduce you the six most common test points in reading comprehension. Once you become familiar with these test points, you will get advantage in speed to come up with the right answer choice. 1. Comparison Words or phrases: like, unlike, in contrast to, similarly Question Type: recall question, inference question Here is an example: The fact that reducing price can generate a competitive advantage for a company does not mean that every reduction in price will create such an advantage. Price reduction, like improvement in service, must be balanced against other types of efforts on the basis of direct, tangible benefits such as increased revenues. If a company is already effectively on a par with its competitors because it provides product at an acceptable price and keeps customers from leaving at an unacceptable rate, then reduction in price may not be effective, since price is not necessarily the deciding factor for any customer in any situation. This truth was not apparent to managers of one operating system software vendor, which failed to improve its competitive position despite its attempt to reduce price. The software managers did not recognize the level of customer inertia that arises from the inconvenience of switching operating system. Nor did they analyze their reduction in price to determine whether it would attract new customers by producing a new standard of price that would excite customers or by proving difficult for competitors to copy. Sample question According to the passage, reduction in price are comparable to improvement in service in terms of the (A) tangibility of the benefits that they tend to confer (B) increased revenues that they ultimately produce

43

(C) basis on which they need to be weighed (D) insufficient analysis that managers devote to them (E) degree of competitive advantage that they are likely to provide To answer this question, first locate the question to the second sentence of the passage. "Price reduction, like improvement in service, must be balanced against other types of efforts on the basis of direct, tangible benefits such as increased revenues." In other words, they are comparable based on which they need to be weighed. Therefore, C is the correct answer.

44

2. Example & Listing Words or phrase: such as, as well as, for example, for instance Question type: Listing, Exampling Let's look at a sample question for the same passage. The fact that reducing price can generate a competitive advantage for a company does not mean that every reduction in price will create such an advantage. Price reduction, like improvement in service, must be balanced against other types of efforts on the basis of direct, tangible benefits such as increased revenues. If a company is already effectively on a par with its competitors because it provides product at an acceptable price and keeps customers from leaving at an unacceptable rate, then reduction in price may not be effective, since price is not necessarily the deciding factor for any customer in any situation. This truth was not apparent to managers of one operating system software vendor, which failed to improve its competitive position despite its attempt to reduce price. The software managers did not recognize the level of customer inertia that arises from the inconvenience of switching operating system. Nor did they analyze their reduction in price to determine whether it would attract new customers by producing a new standard of price that would excite customers or by proving difficult for competitors to copy. The discussion of the operating system software vendor last paragraph serves which of the following functions within the passage as a whole? (A) It describes an exceptional case in which reduction in price actually failed to produce a competitive advantage. (B) It illustrates the pitfalls of choosing to reduce price at a time when business strategy is needed more urgently in another area. (C) It demonstrates the kind of analysis that managers apply when they choose one kind of business strategy over another (D) It supports the argument that strategies in certain aspects are more advantageous than strategies in other aspects. (E) It provides an example of the point about reduction in price made in the first paragraph. Clearly, the author intends to prove his position that reduction in price does not necessarily generate competitive advantage. E is the correct.

45

3. People, Date & Place Phrase: in the nineteenth-century, prior to mid-1970's, Snyder proposed that. Question: inference question, main idea question Indian firms have achieved the highest levels of efficiency in the world software outsourcing industry. Some researchers have assumed that Indian firms use the same programming languages and techniques as Chinese firms but have benefited from their familiarity with English, the language used to write software code. However, if this were true, then one would expect software vendors in Hong Kong, where most people speak English, to perform not worse than do Indian vendors. However, this is obviously not the case. Other researchers link high Indian productivity to higher levels of human resource investment per engineer. But a historical perspective leads to a different conclusion. When the two top Indian vendors matched and then doubled Chinese productivity levels in the mid-eighties, human resource investment per employee was comparable to that of Chinese vendors. Furthermore, by the late eighties, the amount of fixed assets required to develop one software package was roughly equivalent in India and in the China. Since human resource investment was not higher in India, it had to be other factors that led to higher productivity. A more fruitful explanation may lie with Indian strategic approach in outsourcing. Indian software vendors did not simply seek outsourced contract more effectively: they made aggressive strategic in outsourcing. For instance, most software firms of India were initially set up to outsource the contract in western countries, such as United States. By contrary, most Chinese firms seem to position their business in China, a promising yet under-developed market. However, rampant piracy in China took almost 90 percents of potential market, making it impossible for most Chinese firms to obtain sufficient compensation for the investment on development and research, let alone thrive in competitive environment. The author suggests that if the researchers of India mentioned in paragraph 1 were correct, which of the following would be the case? (A) The computer used in India software firms would be different from the computer used in China firms. (B) Indian engineers would be trained to do several different programming jobs. (C) Familiarity with English language would not have an influence on the productivity levels of engineers. (D) The engineers in India-run firms would have lower productivity levels if they have a poor command of English.
46

(E) The production levels of India-run firms located in the China would be equal to those of firms run by China firms. If the researchers are correct, then the familiarity with English determines the productivity of engineers. That is, if the engineers in India-run firms have a poor command of English they would have lower productivity levels, as stated in choice D.

47

4. Words of Attitude and Transition Word: Correctly, qualified, do (does, did), may (might), correctly, first (second, third) Question Type: Tone Question, Main Topic Question, Structure Question The fact that reducing price can generate a competitive advantage for a company does not mean that every reduction in price will create such an advantage. Price reduction, like improvement in service, must be balanced against other types of efforts on the basis of direct, tangible benefits such as increased revenues. If a company is already effectively on a par with its competitors because it provides product at an acceptable price and keeps customers from leaving at an unacceptable rate, then reduction in price may not be effective, since price is not necessarily the deciding factor for any customer in any situation. This truth was not apparent to managers of one operating system software vendor, which failed to improve its competitive position despite its attempt to reduce price. The software managers did not recognize the level of customer inertia that arises from the inconvenience of switching operating system. Nor did they analyze their reduction in price to determine whether it would attract new customers by producing a new standard of price that would excite customers or by proving difficult for competitors to copy. The passage suggests that operating system software managers failed to consider whether or not the price reduction mentioned last sentence (A) was too complicated to be easily described to prospective customers (B) made a measurable change in the experiences of customers purchasing (C) could be sustained if the number of customers increased significantly (D) was an innovation that competing vendors could have imitated (E) was adequate to bring the vendors general level of price to a level that was comparable with that of its competitors The passage following failed to describes the failure. The best choice is D, which is stated in the last sentence.

48

5. Counter-evidence Indicators Counter-evidence words warn that the author is about to either make a U-turn or introduce a counter-premise (concession to a minor point that weakens the argument). But Despite In contrast Although Nevertheless Even though However Nonetheless Yet Except

Counter-evidence words mark natural places for questions to be drawn. At a pivotal word, the author changes direction. The GMAT writers form questions at these junctures to test whether you turned with the author or you continued to go straight. Rarely do the GMAT writers let a pivotal word pass without drawing a question from its sentence. As you read a passage, note the pivotal words and refer to them when answering the questions. Example China as a nation faces two major financial problems. First, eighty-four percent of state-owned enterprises do not generate profit. Government failed to make money from such business. Rather, it has to appropriate substantial funds to these enterprises in order to prevent them from going bankrupt and thus resulting in high unemployment rate. Second, 203 million of civilians in countryside will not be able to gain pension after they retire due to the limited budget of government. I would like to make an outrageous suggestion that would at one stroke generate finance earnings and provide funds for civilians retirement. I would propose that government sells its holdings in state-owned enterprises on the open market. Such sales would provide substantial funds for village civilians pension. At the same time, they could cut down financial burden on these state-owned enterprises. You might object that government would be deprived of the opportunity to share its enterprises profit if someday they make money. I agree. Sell holdings of enterprises that would never generate profit. But, you might reply, every enterprise that competes on the market has potential. Here we part company. Theoretically, you may be correct in claiming that every enterprise has the potential to make money. Practically, you are wrong. I refer to the thousands of state-owned enterprises that are not likely to make money. These companies are 100 percent held by the nation as a whole. Government officials are appointed as the chairman, CEO and president. The management was not responsible for the public interest, but for the nation as a whole. If there is no significant loss in business,
49

they will soon be promoted back to the higher level position in government. If their companies perform great, these executives receive direct money compensation. However, their salary, when combined with such compensation, will be much less than the amount they would earn if were in private company. It would be unrealistic to suggest that village civilians would have sufficient funds if governments shares were sold on the open market. But the demand for compensating the state-own enterprises would be substantially reduced. The author anticipates which of the following initial objections to the adoption of his proposal? (A) Government will not be able to sell its holdings with state-owned enterprise. (B) The ability of governments to control the national economy will be weakened if state-owned enterprises are sold to private owners. (C) It is impossible to find enterprises that will never generate profit. (D) The poor performance of state-owned enterprises will continue. (E) The countryside civilians are sill unable to seek financial support from government. In the third paragraph, the author illustrates a possible objection to his proposal. The opposite views are government would be deprived of the opportunity to share its enterprises profit if someday they make money and every enterprise that competes on the market has potential. Choice C presents one of them, and is the correct answer.

50

6. Special Punctuation Punctuation: Quotation, Parentheses, Dash Question type: Recall question, Inference Question The cutting-edge science is ringing alarm bells. Avian flu virus picked up by pigs can swap genetic materials with another flu virus already in the pig and become a new, hitherto unknown flu virus for which no person, no animal has preexisting immunity. The kind of virus causes a pandemic because it spreads from human to human. If you took a peek into history, it turns out that previous influenza pandemics have similar scenarios. The greatest influenza pandemic in 1918 caused more than 20 million deaths of soldiers stationed in France. The last influenza pandemic was in 1968, known as the Hong Kong flu (H3N2). Thousands of deaths and millions were infected worldwide. The other examples are the Nipah virus and Japanese Encephalitis virus, which find pigs to be good hosts. With JE, the virus circulates in the blood of infected pigs. When infected pigs are bitten by Culex mosquitoes, the virus replicates in the mosquito's gut. The next time the mosquito bites a human, the virus is passed on. The pig doesn't get sick as such. The Nipah virus causes pneumonia symptoms in pigs. In humans, it causes encephalitis, and humans catch it only with direct contact with infected pigs. Symptoms range from mild headache to permanent brain damage, and can be fatal. It's merely a phenomenon of nature that the pig is the "mixing vessel" for the new germ. But make no mistake, the pig is not the villain, neither is the chicken. It's actually us, and our horrible farm practices, outdated agricultural policy and, most of all, reckless disregard of our ecology and environment. "Hygiene and management can control what eventually happens," says Lam. "Good farming practice will prevent serious outbreaks and infection to humans." Despite knowing that, animal diseases and the possibility of transmission to humans are becoming quite alarming. Of the 35 new emerging diseases in the last 20 years, more than 70 per cent involved animals. In fact, what we may have done is unwittingly create the perfect launch pad for an influenza pandemic that will likely kill large numbers of people across the globe. Although scientists say it's impossible to predict the odds that the virus will alter its genetic form radically enough to start leaping from human to human, the longer H5N1 is out there killing chickens, the higher the chances are. What does the author mean by describing the pig as mixing vessel? (A) Pig is the place where various viruses reside.

51

(B) Pig is the pot in which viruses swap genes and become new, deadly germs. (C) Viruses are mixed inside the body of pig. (D) New germs come to the body of pig and reside there. (E) Pig attracts viruses. The question requires you to determine the meanings of mixing vessel. At the beginning of the passage, the author states that Avian flu virus picked up by pigs can swap genetic materials with another flu virus already in the pig and become a new, hitherto unknown flu virus for which no person, no animal has preexisting immunity. The kind of virus causes a pandemic because it spreads from human to human. In other words, pig is the pot in which viruses swap genes and become new, deadly germs. Therefore, the correct answer is B.

Review
One principal Don't rely on memory or on daily life experiences. Answer all the questions on the basis of what is stated or implied in the passage. Two Styles Presentation is to deliver an idea that the author will agree or partially agree. Argumentation is to develop two ideas or systems and then point out why one is better than the other (contrasting) or just simply refute both of them and developed the author's own idea. Three Subjects There are three major subjects that a typical GMAT reading passage may discuss about: natural science, social science and business subject. Four-step Procedure Use the four-step process to read a passage. First dissect the first paragraph, then skim the passage, get the main idea and diagram the organization of the passage. Five Types of Question There are five types of question you may encounter in real GMAT test: main idea question, recall question, inference question, critical reasoning question and the difficult-to-locate question. Six Test Points The most common six test points are comparison, exemplifying, people, special punctuation, counter-evidence and mood words.

52

Chapter 2 Sentence Correction


Introduction
Sentence Correction on the test day About 17 of the 41 Verbal section questions are Sentence Correction. And in the first 10 question, there are about 5 sentence correction question. This is why Sentence Correction is so important to your GMAT performance. The directions on the test day for this part look like this: Directions: The following questions consist of sentences that are either partly or entirely underlined. Below each sentence are five versions of the underlined portion of the sentence. Choice (A) is a copy of the original version. The four other answer choices change the underlined portion of the sentence. Read the sentence and the five choices carefully and select the best version. You are expected to pick up one choice that will make the whole sentence clear, concise and free with grammar errors. Choice A is always the same as the original underlined, so you only have to read the other four choices. What does the Sentence Correction test? The Sentence Correction section tests the correctness of English grammar, including subject-verb agreement, modification, pronouns, idioms, and etc. The GMAT also tests the effectiveness of expression, such as parallelism, and logicality. Therefore, the best answer should be both exact and clear, without ambiguity, redundancy, or awkwardness. The GMAT does not test mechanics concerns, such as punctuation, capitalization, misspelling, etc. You should never spend time on figuring out whether a word is misspelled, or whether a comma is the right punctuation here. Rather, you should focus on the grammar usage and standard written English. Now, let's look at an example to see what a sentence correction question could be. Satisfied by the strong performance on GMAT test, it was decided by Peter to give himself a two-week rest. A. B. C. it was decided by Peter to give himself a two-week rest Peter decided to give himself a two-week rest a two-week rest was given by Peter to himself
53

D. E.

Peters decision was to give himself a two-week rest it was decided that Peter give himself a two-week rest

Three-step method
A. Read the complete sentence Read the whole sentence to get an idea about the basic meanings. Dont read only the underlined part of the sentence. If you simply read the underlined, you are probably following the trap set by the test maker. Example Satisfied by the strong performance on GMAT test, it was decided by Peter to give himself a two-week rest. B. Figure out what the question is testing Before you read the answer choices, spend several seconds on figuring out what the question is testing. If you have no idea, you may look at the different answer choices and see what the changes have been made, so as to figure out the possible errors in the sentence. Satisfied by the strong performance on GMAT test, it was decided by Peter to give himself a two-week rest. A. B. C. D. E. it was decided by Peter to give himself a two-week rest Peter decided to give himself a two-week rest a two-week rest was given by Peter to himself Peters decision was to give himself a two-week rest it was decided that Peter give himself a two-week rest

C. Eliminate wrong choices The process of elimination is extremely useful in Sentence Correction section. Eliminate it once you find one error, until you are left with only one or two choices. If you are unable to eliminate four choices, take a second look at the original sentence. Some questions test more than one grammar knowledge. If you still fail, you may just make a guess. Satisfied by the strong performance on GMAT test, it was decided by Peter to give himself a two-week rest. A. it was decided by Peter to give himself a two-week rest
54

B. C. D. E.

Peter decided to give himself a two-week rest a two-week rest was given by Peter to himself Peters decision was to give himself a two-week rest it was decided that Peter give himself a two-week rest

The correct answer is B because it is Peter who was satisfied. In choice B, Peter appears as the subject. Choices A, C, D and E are incorrect because they used it, a two-week rest, Peters decision, and it as the sentence subject. In fact, the participial phrase beginning with satisfied should modify the subject of the main clause in order to follow the grammar rule. Fourteen types of errors Based on our research, the following 14 types of errors represent 96% of Sentence Correction questions. You should become sensitive to these errors.

55

Section 1: Subject-Verb Agreement


The subject and verb must agree both in person and number. If the subject is plural, then the verb must be plural too. If the subject is singular, then the verb must also be singular. Intervening modifier has no effect on subject-verb agreement. If a singular subject is accompanied by phrases or clauses, it remains singular. Subject that contains and is plural. The subject that contains or can be either plural or singular depending on the number of the last item. When the subject and verb are reversed, they still must agree in both number and person. Note: 1) The following nouns are singular: work, happiness, poverty, honesty, faith, time, visibilitymilk, tea, cotton, petrol 2) Singular and plural nouns with same form: bison, cattle, sheep, fish, aircraft, means, series, Chinese, Japanese 3) Subject nouns ending with ics are singular: economics, statistics, 4) Verb phrases as a subject is singular. Preparing for the TOEFL is not an easy task. Example Since 1999, the number of internet websites with the domain name ending with .com have grown from 62 million to nearly 78 million. A. B. C. D. E. have grown from 62 million to nearly 78 million are growing from 62 million to nearly 78 million grew from 62 million to nearly 78 million grow from 62 million to nearly 78 million has grown from 62 million to nearly 78 million

E is the correct answer. In choice A, the plural verb have does not agree with the singular subject number. Choices B and D commits the same fallacy by using plural verbal phrase are growing and grow respectively. B, C, and D also misused the verbal tense which should be present perfect.

56

Section 2: Verb Time Sequences


Faulty verb tense is common on GMAT test. But it is easy to solve if you become familiar with verb tense rules. A verb has four principal parts: present tense, past tense, past participle and present participle. The present tense is used to express present tense or general truths. The past tense is used to express past tense. The past participle is used to form the present perfect tense, past perfect tense, or future perfect tense. The present participle is used to form the present progressive tense, the past progressive tense, or the future progressive tense. Keep alert when several verbs appear. If several events happened at different times, choose one as the basic in time sequence. Eighty percent of notebook computers that were sold in United States last year were manufactured in China, a country that has the largest population in the world. A. Eighty percent of notebook computers that were sold in United States last year were B. Eighty percent of notebook computers that were sold in United States last year had been C. Eighty percent of notebook computers that were sold in United States last year have been D. Last year eighty percent of notebook computers were sold in United States that have been E. Last year eighty percent of notebook computers that were sold in United States had been In using only one verb tense, were, choice A fails to indicate that the computers were manufactured before sold. Choices C and D use the present perfect tense incorrectly, saying in effect that the computers have been manufactured after they were sold last year. Choice E suggests that the manufacturing of the notebook computers, rather than selling, occurred last year, thus making the sequence of events unclear. Only B uses verb tenses correctly to indicate that manufacturing of the computers was completed prior to the selling.

57

Section 3: Modification
In GMAT grammar, a modifier can be an adjective, adverb, phrase, an appositive or even a clause. In the following passage, we will introduce each of them. As a general rule, a modifier should be placed as close as possible to what it modifies. A. Adjective or adverb as a modifier An adjective can modify a noun, but an adverb can not. An adverb is often used to describe the extent or degree of an adjective or a verb. Example According to a report from Anderson Accounting, the gross sales of General Movies in 2002 were $86 millions as many as their expected revenues. A. as many as their expected B. more than their expected C. as many as their excepted D. more than their expectedly E. as many as their expectedly Choices A, C, and E do not state the comparison logically. The expression as many as indicates equality of quantity, but the sentence indicates that the gross sales exceed the expected revenues by $86 millions. In B, the best choice, more than makes this point of comparison clear. B also correctly uses the adjective expected, rather than the adverb expectedly used in D and E, to modify the noun phrase revenues. B. Clause as a modifier A clause beginning with that, which or where modifies the words or phrases nearest to it. Here, "which" initiates a clause modifier and can not be used to refer to a sentence. Example The current downturn in the U.S. economy is encouraging many young professionals to return to school, which doubles to twice the number of applicants five years ago. A. B. C. which doubles to twice the number of applicants five years ago doubling to twice the number of applicants five years ago which doubles to twice the number of applicants that were five years ago
58

D. E.

doubling to twice the number of applicants five years before which doubles to twice the number of applicants five years before

The pronoun which should be used to refer to a previously mentioned noun, not to the idea expressed in an entire clause. In A, C, and E, which seems to refer to a vague concept involving young professionals returning to school, but there is no specific noun, such as return, to which it can refer. B and D use the correct participial form, doubling, to modify the preceding clause, but D, like A, uses five years before rather than five years ago, a phrase that is more idiomatic in context. B, therefore, is the best answer. C. A long phrase as a modifier When a long phrase initiates a sentence, make sure that it modifies the subject of the sentence. Example Using the KB833330, a new virus known as Bagle can be blocked outside the Local Area Network. A. Using the KB833330, a new virus known as Bagle can be blocked outside the Local Area Network. B. A new virus known as Bagle can be blocked outside the Local Area Network, using the KB833330. C. Blocking a new virus known as Bagle outside the Local Area Network, an engineer can use the KB833330 by an engineer D. Outside the Local Area Network, a new virus known as Bagle can be blocked using the KB833330 by an engineer E. Using the KB833330, an engineer can block a new virus known as Bagle outside the Local Area Network. Choice A presents a dangling modifier. The phrase beginning the sentence has no noun that it can logically modify and hence cannot fit anywhere in the sentence and make sense. Coming first, it modifies a new virus, the nearest free noun in the main clause; that is, choice A says that a new virus are using the KB83330. Choice B contains the same main clause and dangling modifier, now at the end. Contrary to intent, the wording in choice C suggests that engineers can use the KB833330 after they block a new virus. In choice D the phrase using ... the KB833330 should follow engineer, the noun it modifies. Choice E is best.

59

D. Appositive as a modifier Like a clause, an appositive can also serve as a modifier. In most cases, appositive is separated by a comma. The appositive should be in the same number as modifiee. Example The GMAT math section consists of 37 questions, each question a test of a certain math concept. A. B. C. D. E. each question a test on a certain math concept all the questions a test on a certain math concept all the questions are tested on a certain math concept every question is tested on a certain math concept each question is tested on a certain math concept

Choice A is best: the appositive terms question and test, both singular, agree in number; both also agree with rule on sentence structure. In C, D, and E choices, Runs-on sentence is committed.

60

Section 4: Parallelism
Similar elements must be expressed in similar form: all nouns, all infinitives, all gerunds, all prepositional phrases, or all clauses must agree. Test writers often use a parallel structure for dissimilar elements. In the case you are not sure which form should be used, the form of the second element of the series determines the form of all subsequent elements. Example According to a survey, a company president typically spends 60 percent of his or her time on communicational activities, such as answering the calls, communicating with clients and to speak on the meetings. A. B. C. D. E. communicating with clients and to speak communicating with clients and speaking to communicate with clients and speak to communicate with clients and to speak to communicate with clients and speaking

Because the verb phrases used to describe the communicational duties are governed by the phrase communicational duties such as, they should each be expressed in the present participial (or "-ing") form to parallel answering. Choices A, C, D, and E all violate parallelism by employing infinitives (to...) in place of participial phrases. Only B, the best answer, preserves the sense of the original, uses the correct idiom, and observes the parallelism required among and within the three main verb phrases.

61

Section 5: Pronoun
Pronouns must agree with their antecedents in both number and person. Subject pronoun must be used as a subject. Object pronoun must be used as an object. A pronoun must clearly stand for a noun. If a pronoun follows two nouns, it is often unclear which of the nouns the pronoun refers to. This is the most common error on the GMAT. The word "which" introduces non-essential clauses and "that" introduces essential clauses. "Who" refers to individuals; "that" refers to a group of persons, class, type, or species. Subject I We You You He She It they Example The best way for an IT professional to protect data is to periodically back it in a pre-formatted disc. A. B. C. D. E. to periodically back it in a pre-formatted disc if it is quickly backed in a pre-formatted disc for it to be backed periodically in a pre-formatted disc if the data is periodically backed in a pre-formatted disc to have them periodically backed in a pre-formatted disc Object Me Us You You Him Her It Them Possessive (adjective) my our your your his her its their Possessive (nouns) mine ours yours yours his hers its theirs myself ourselves yourself yourself himself herself itself themselves Self Singular or Plural Singular Plural Singular Plural Singular Singular Singular plural

For parallelism, the linking verb is should link two infinitives: The only way to salvage ... is to back. Choice A begins with an infinitive, but the plural pronouns I do not agree with the plural noun data. Choices B, C, and D do not begin with an infinitive, and all present pronoun errors: the singular pronouns cannot grammatically refer to data. The best choice, E, has parallel infinitives.

62

Section 6: Comparisons
The comparison should be both logically similar and grammatically parallel. There are typically three types of comparisons: quality comparison, quantity comparison and analogy.

Types

of

Words of Comparison

Indicators Comparison

of

Comparison Quality like/unlike in contrast to rather/other than Quantity Analogy less/more than just as as much as so too the same as so as

that/those

of,

was/is/were/are, did/do/does

1. Quality Comparison If the comparison begins with like, unlike, rather than, in contrast to, not but, it is a quality comparison. This comparison is common, but easy in sentence correction. All you have to do is to check if the item following the word of comparison is comparable to the subject of main sentence. Example Unlike a corporation, which pays tax based on its related revenues, a fixed amount of tax is paid by a sole ownership business. A. a fixed amount of tax is paid by a sole ownership business B. with a sole ownership business a fixed amount of tax is paid C. a sole ownership is paid a fixed amount of tax D. for a sole ownership business a fixed amount of tax is paid E. a sole ownership pays a fixed amount of tax Choice E, the best answer, correctly uses a parallel construction to draw a logical comparison: Unlike a corporation,..., a sole ownership business.... Choice A illogically compares a corporation, an entity, with a fixed amount of tax, money. In choice C, a sole ownership business can not be paid for tax. Choices B and D are syntactically and logically flawed because each attempts to compare the noun corporation and a prepositional phrase: with a fixed amount of tax. Choices B and D are also imprecise and awkward. Finally, choice E is the only option that supplies an active verb form, pays to parallel pays.

63

2. Quantity Comparison Typical comparisons in this kind are introduced by such idioms as more/less than, the same as, as many as. The errors in quantity comparison are more difficult to identify since the sentence that contains quantity comparison is generally long. The following two procedures can be taken to tackle quantity comparison: Check if the idiom or phrase is correct and complete Check if the introductory word has been presented to initiate the comparison

Example In addition to having more employees than UT StartCom, the employees in GenericSart are higher educated than those in UT StartCom, with more graduate students. A. B. C. D. E. the employees in GenericSart are higher educated than those in GenericStart has higher educated employees than those do the employees in GenericStart are higher educated than those are in GenericStart employees are higher educated that those are in GenericStart has higher educated employees than

In this sentence, the initial clause modifies the nearest noun, identifying it as the thing being compared with UT StartCom. By making employees the noun modified, choices A, C, and D illogically compare UT StartCom with employees and claim that the employees in GenericStart has higher educated employees than UT StartCom does. B, the best choice, logically compares UT StartCom to GenericStart by placing the noun GenericStart immediately after the initial clause. B also uses those to refer to employees in making the comparison between the employees of UT StartCom and GenericStart. Choice E needs either those in or do after UT StartCom to make a complete and logical comparison. 3. Analogy While analogy is a logical concept, it is also used in Sentence Correction to draw a comparison between two similar things. Like comparison, analogy should also be logical and grammatical. Example The gravity will apply the same to an airplane flying in the air as a ship floating on the water. A. B. air as a air as to a
64

C. D. E.

air; just as it would to a air, as it would to the air; just as to the

B, the best choice, uses the idiomatic and grammatically parallel form the same to X as to Y.

65

Section 7: Choice of Word


Sometime, the test-takers try to puzzle you by using some similar words with different meanings. In choosing a choice, make sure it doesn't contain the wrong word. Affect/Effect: Effect is a noun meaning "a result." Example: Increased fighting will be the effect of the failed peace conference. Affect is a verb meaning "to influence." Example: The rain affected their plans for a picnic. Fewer/Less: (number/amount) Use fewer when referring to a number of items. Use less when referring to a continuous quantity. Example: In the past, we had fewer options. Example: The impact was less than what was expected. Example A report by Business Weekly indicated that the number of money invested by companies in Business and Research in 2003 was twice that in 2002. A. the number of money invested by companies in Business and Research in 2003 was B. the number of money invested by companies in Business and Research in 2003 were C. the number of money invested by companies in Business and Research in 2003 are D. the amount of money invested by companies in Business and Research in 2003 were E. the amount of money invested by companies in Business and Research in 2003 was Choices A, B, and C are flawed because the uncountable noun money should be modified by amount rather than number. In addition, B, C, and D incorrectly use the plural verb were or are with the singular noun money. Choice E, the best answer, is both grammatically correct and concise.

66

Section 8: Idioms
We have summarized most of the idioms tested on GMAT sentence corrections. By memorizing the following lists, you should be able to solve most the idioms questions on the test day.

act on account for at least a great deal

ability to do ask for at one time a number of

according to as far as is known a cluster of a minimum of

bare of begin to be adopted to be based on be characterized by be considered to be be depend on be divided into be expose to be filled with be full of be known as be made of be originated from be resistant to be similar to be valuabel for

belong to benefit from be associated with be beneficial to be composed of be credited with doing be derived from be engaged in be familiar with be forced to do be inclined to be known for be noted for be rich in be subjected to be suited for be viewed as

begin doing be able to be appreciated for be capable of be concerned with be dedicated to doing be destined to be equal to be famous for be found in be involved in be made from be obtained from be related to be supposed to be typical of be woven from
67

break away from by means of

bring about

bring to light

close to consist of concentrate on comment on come from carry out

change into contrary to convert t into come into contact with come into contract with

coincide with contribute to combine with

date back to demand for devote to do no harm to

deal with depend on differ from do/deal with

dedicate to doing deter sb. from doing draw from due to + n.

excel in/at

feed on

focus attention on

68

give off grant sb. sth.

give way to

give up doing

interest in in connection with in nature in relation to in the future

interfere wit in danger of in honor of in response to

in addition to in history in comparison with in spite of

join with

lead to

live in

look for

make up more than

meet one's goal

mingle with

69

native to

no more than

not more than

on account of

on behalf of

participate in play a key role

pay for protect from

permit sb. to do prohibit..from

range from to rely on result from

rank first among regardless of run for

refer to rest on

settle down shield from spend doing start to succeed in doing

sever as so as to spread to strive to

share with specialize in doing start doing substitute for

70

take place the ratio toof to togeter with

take charge of the use of to a great extent

tend to think of transform into

use up

make use of

vary in

W
warm sb. of sth.

71

Section 9: Sentence Structure


There are limited types of sentence used in English language. The five of them are Declarative (simple statement): (Subject first) Mary (then verb) loves (then object) turnips. Interrogative (a question): Do you need help identifying a question? Imperative (command): Do as I say. Exclamatory (exclamation): What exciting it is! Conditional (condition expressed): If I pass the exam, I intend to move to Arizona. Sentences are also classified as simple and complex. Complex sentences contain phrases and clauses that transform the "I like candy" sentences of your childhood to the more sophisticated constructions of adult speech. The most common errors for sentence structure is the RUN-ON sentence, a sentence that lacks of proper conjunctions. Wrong: There is agreement among United States voters that there is waste in government and that the government as a whole spends beyond its means, it is difficult to find broad support for a movement toward a minimal state. Correct: However much United States voters may agree that there is waste in government and that the government as a whole spends beyond its means, it is difficult to find broad support for a movement toward a minimal state. Example However much people may agree that there is substantial corruption in Chinese government and that government does its endeavor, it is difficult to keep the problem under control. A. B. C. D. E. However much United States voters may agree that Despite the agreement among people to the fact Although people agree Even though people may agree There is agreement among people that

A is the best choice. Choices B, C, and D incorrectly omit that after agree; that is needed to create the parallel construction agree that there is substantial corruption . . . and that the government... . Choice E, though it retains that, is grammatically incorrect: because E starts with an independent rather than a subordinate clause and separates its two independent clauses with

72

a comma, it creates a run-on sentence with no logical connection established between the halves. In B, the agreement ... to the fact is unidiomatic, and B, C, and E alter the sense of the original sentence by saying that voters agree rather than that they may agree.

73

Section 10: Subjunctive Mood


When using a subjective verb, the verb following the clause should be in present tone. Also, the conjunction "that" should not be removed. The most common subjunctive verbs include:

require demand request suggest rule

Example Given that most Misubishi cars on the road have some problems with tyre, the Misubishi Company yielded to hundreds of unsatisfied buyers requesting that it should call back the cars on the market. A. requesting that it should B. requesting it to C and their request to D. who requested that it E. who request it to Choice D, the best answer, uses the grammatically correct expression requested that it call back, in which requested that it is followed by the subjunctive verb call. Choice A incorrectly uses should call rather than call: requesting that already conveys the idea of "should," and at any rate a modal auxiliary verb, such as should or must, cannot grammatically follow the expression requested that. Similarly, B and E use the ungrammatical expression requesting/requested it to. In C, the expression yielded to... buyers and their demand to call... unnecessarily states that the company yielded to the buyers as well as to their request. This expression also fails to specify that the company is expected to call back the cars on the market.

74

Section 11: Ambiguity


Some answer choices may be correct in grammar, but not the right one. If you are unable to eliminate four choices according to grammar usage, you should look at whether it expresses clearly without ambiguity. Pay special attention to choices that contain "if" since this conjunction has two meanings: 1. 2. In the event that: If you get well prepared for GMAT, you will score higher. Whether: I am unsure if you take the GMAT.

So, always avoid the usage of if when it serves as the meaning of whether. Example Opposites of privatization in China consider state-owned business to be an integral part of national treasury and question if privatization could weaken the ability of the country to adjust national economy. A. B. C. D. E. to be an integral part of the criminal justice system and question if as an integral part of the national treasure and they question if as being an integral part of the national treasure and question whether integral part of the national treasure and question whether are an integral part of the national treasure, and they question whether

When consider means "regard as," as it does in this sentence, its object should be followed immediately by the phrase that identifies or describes that object. Thus, to be in A, as in B, and as being in C produce unidiomatic constructions in the context of the sentence. Also, although (/and whether can be used interchangeably after some verbs, question if, which appears in A and B, is unidiomatic, and they in B is unnecessary. E also contains the unnecessary they, and it uses the ungrammatical construction consider... facilities are. Grammatically and idiomatically, sound D is the best choice.

75

Section 12: Redundancy


As I said at the beginning of this Chapter, Sentence Correction not only tests your knowledge of correct grammatical usage, but also your sense of clear English writing. Your selected answer should express the intended meaning of the original sentence as precisely as possible, while avoiding unnecessarily wordy constructions. The following phrases illustrate typical wordy construction you will encounter in some fluff answer choices: Wordy: more than or as much as Concise: at least as much as Wordy: a noun that is (are) adj. Concise: an adj. noun Example At least as much as 204 millions or more people in the world earn fewer than $10 per day. A. B. C. D. E. At least as much as 204 millions or more people in the world earn fewer than At least as much as 204 millions or more people in the world earn less than More than 204 millions people in the world earn fewer than More than 204 millions people in the world earn less than There are at least 204 millions or more people earn less than

D, the best choice, is idiomatic, clear, and concise. Both A and B incorrectly use much rather than many to describe the countable noun others. Even if this error were corrected, though, A and B would still be wrong. Because more than x necessarily includes the sense of at least as many as x. it is redundant and confusing to use elements of both expressions to refer to the same number of people. In A and C, fewer is misused.

76

Section 13: Awkward


Sometime, you will find the underlined difficult to understand. You can't tell which part of the underlined has problem. Instead, you just feel A would not be the right choice. Here, you should consider whether the expression is awkward. The following expression is awkward, and will never be the right choice. A long phrase as a modifier while without a comma as a proper pause. There be + abstract noun (such as conversion, relation).

Example Linguists conclude that the more the non-Chinese use Chinese, their intellectual advantage is greater in skills underlying spatial arranging ability. A. their intellectual advantage is greater in skills underlying spatial arranging ability. B. their intellectual advantage is the greater in skills underlaying spatial arranging ability. C. the greater their intellectual advantage in skills underlying spatial arranging ability. D. in skills that underlay spatial arranging ability, their intellectual advantage is the greater E. in skills underlying spatial arranging ability, the greater intellectual advantage is theirs The best choice is C. The phrase the more the non-Chinese should be completed by a parallel phrase that begins with a comparative adjective and a noun phrase, as in the greater their... advantage. Only C correctly completes the structure with a parallel phrase. Choices A. B, D, and E present structures that are unwieldy and awkward in addition to being nonparallel, and that state the relationship between language use and skills development less clearly than C does. Also, underlaying in B and underlay in D are incorrect; the meaning of this sentence requires the present participle of "underlie," underlying, as a modifier of skills.

77

Section 14: Logicality


Like the rule of comparison, you can not say psychopath is a person who. The proper expression should be psychopath refers to someone who. Example In 2003, the term SARS went deep into everyones mind in South-eastern Asian Area, and in pathology it is severe acute respiratory syndrome. A. B. C. D. E. it is severe acute respiratory syndrome it is someone suffered from severe acute respiratory syndrome they are severe acute respiratory syndrome it refers to severe acute respiratory syndrome it is in reference to severe acute respiratory syndrome

Answer: D In choices A and B, the pronoun it simultaneously refers forward to severe acute respiratory syndrome and backward to the term "SARS" As a result, the sentence asserts illogically that the term is actually a kind of illness rather than a word referring to a kind of illness. Choice C repeats this fault and adds an error in agreement: they (plural) does not agree in number with the term (singular). E omits a main verb, such as applied, that, in grammatical context here, is required after is. In choice D, the best answer, the verb refers is correctly used after it, and the alignment of pronouns and antecedents is both logical and grammatical.

78

Summary: Finding an error step-by-step in Sentence Correction


With the above fourteen major errors in mind, you are able to eliminate the incorrect choices in a typical sentence correction question. Below, I will introduce the step-by-step procedure that will help identify whether the underlined follows the rule of Standard English expression.

Step 1: Is the original sentence correct in the sentence structure? Pay attention to the Runs-on Sentence.

Step 2: If the underlined contains a verb, check whether the subject and verb agree both in number and person. Also the tense of the verb should agree with the time sequence.

Step 3: If the underlined contains pronouns, the pronouns must agree with their antecedents in both number and person. If it contains several pronouns, the pronouns should refer clearly to their antecedents.

Step 4: If the underlined contains a word of comparison, make sure the comparison is logical, clear and paralleled.

Step 5: If the sentence contains a modifier of phrase, check whether it modifies the subject clearly.

Step 6: If the underlined contains a clause, make sure the introductory pronouns (where, which) is correct. If the sentence contains a subjunctive, is that omitted? verb in present tense?

Step 7: If the underlined contains the conjunction or, and, check if similar elements in a list should be in similar form.

Step 8: If the underlined contains an idiom, check if the idiom is correct and complete. Step 9: If the underlined conforms to all the above grammar rules, check if the sentence is clear, concise, and without awkward.

79

Chapter 3 Critical Reasoning


On the test day, you will encounter 41 verbal questions, 10 out of them are Critical Reasoning. To make it comfortable for you to prepare for this subtest, we have designed an interesting "number" course (the name of each strategy begins with a number) to help you learn the test prep strategies. Section 1: Introduction One Definition Four Elements of an Argument Seven Common Fallacies Three-element Rule Two Traps Five Answer Choices Section 2: Six Types of Reasoning Section 3: Eight Types of Question

80

Section 1: Introduction to Critical Reasoning


1. One Definition: Argument Most people call the Critical Reasoning as Argument. An argument is a coherent series of statements leading from a premise to a conclusion. Formula of argument looks something like this: Premise + Example/Reason = Conclusion Depending on the effectiveness of premises and the logicality of reasoning (use of example and reason), an argument can be perfectly true or totally fallacious. For instance, someone may conclude Candidate G will become the next president of United States because in a recent poll, eight out of ten gave votes to him. This poll can be effective if the respondents are representative to the whole electorate, or be ineffective if the respondents are biased sample. However, the critical reasoning question never asks you to determine whether an argument is correct or incorrect. Rather, the question requires you to evaluate the reasoning of an argument. In other word, arguments are designed to test your ability to think logically, not your ability to seek truth. Lets look at a sample argument: Our work proves to be very successful. In the past three years, each of our five clients has experienced the fastest growth of sales in their history. Therefore, if your company meets management problems, do not hesitate to call Sigma & Max, since we are the best management consulting company. Here, the advertiser tried to convince that it is the best management consulting company available and to persuade its potential customer to choose its service. To support its statement, the advertiser cited five of its clients, each of them gained the fastest growth rate in their history.

81

2. Four elements of an argument An argument is a coherent series of statements leading from a premise to a conclusion. One can cite evidences to strengthen an argument or attack its assumption to weaken an argument. Premises + Evidence = Conclusion I. Conclusion How do you identify the authors conclusion? Most often, a conclusion is stated in the last or first sentence in an argument. The conclusion rarely comes in the middle of an argument. Also, you can search for the conclusion indicators that are commonly used to introduce a conclusion. Conclusion Indicators

so consequently conclude that

thus accordingly follows that

therefore hence means that

as a result imply infer that

II. Premise Premise is the fact or reason that the author uses to strengthen his argument. The following are phrases that introduce the premises of an argument. Premise Indicators

because if

since assume the reason is that

for suppose may be derived from

as evidence

on the basis of

in that

III. Counter-evidence Sometimes, the author uses counter-evidence words to argue against his opposite, or concede certain minor points that may weaken his argument. For the latter, the counter evidence is finally refuted by further evidence. Following are some of the most common used counter-evidence indicators.
82

Counter-evidence Indicators

actually even though however

despite nonetheless In spite of

admittedly nevertheless do

except although may

IV. Assumptions Assumptions are those that the author uses to strengthen his argument but leaves it unstated. The assumption is a gap between the premises and the conclusion. In order to evaluation an argument, you always have to find this gap.

83

3. Seven Common Fallacies There are numerous fallacies that you may encounter on the test day. To help you easily identify the logical flaws on the test day, we have summarized the three most common logical fallacies. I. Biased Sample Fallacy The biased sample fallacy is committed when the author draws a conclusion concerning a population based on a biased sample. Here is an argument that commits the fallacy of the biased sample: In a recent survey conducted by Computer Weekly, 93% of the respondents indicated that Computer Weekly is the best computing magazine that they had subscribed. This survey clearly shows that Computer Weekly is the best computing journal available. The data of 93% are drawn from a sample that does not represent the entire electorate. Since the survey was conducted by people who had subscribed Computer Weekly, not all subscribers of computing journals have an equal chance of being included in the sample. In other words, people who had read Computer Weekly might be more likely to have a favorable opinion on Computer Weekly. Therefore, the sample is not representative, and is biased. II. Insufficient Sample Fallacy When someone draws a conclusion about a population based on a sample that is not large enough, he or she commits the insufficient sample fallacy. A typical form would be: 1) X% of A are B. 2) Therefore, A are B. III. Questionable Cause Fallacy This fallacy has the following general form: 1) A happens before B. 2) Thus A is the cause of B. It is an error in reasoning to conclude that one thing causes another simply because one happens before the other. The fallacy committed is that a causal conclusion is being drawn from inadequate evidence. IV. Personal Attack

84

Personal attack, as its name suggests, is a technique to challenge a persons character instead of his opinions. As a matter of fact, there is no correlation between personal character and his position. A sample statement would be: Dont believe him; he had a bad credit. V. Equivocation Equivocation is the use of a word in two or more meanings in an argument. The author tries to prove his statement, but the underlying subject is in a second meaning other than the most commonly used. The average normal infant born in the United States weighs between twelve and fourteen pounds at the age of three months. Therefore, if a three-month-old child weighs only ten pounds, its weight gain has been below the United States average. Here, the flaw is that average weight gain is not the same as average weight. VI. Appeal to authority Someone is appealing to authority when he cites an experts opinion to support his own position. It is obvious that an experts statement can not be always true. Especially if the expert is not specializing in the filed related to the argument, his opinion is very likely to be fallacious. Here is an example. Mr. Kate can definitely solve the management problems of our company because Mr. Kate is a professor in Management. Its true that Mr. Kate is an expert in management research, but not necessarily in real-world management experience. VII. Faulty Analogy Faulty analogy is committed when someone reasons that two things are similar in this way because they are similar in that way. The following argument commits the faulty analogy. Peter and Jennifer were twins. Since Peter like dancing, Jennifer will also like dancing.

85

4. Three-element Rule Known as "Transitive Property" or "Syllogism", the three-element rule is among the most common reasoning methods you can use to solve GMAT Critical Reasoning problems. This rule could be expressed by the formula: If Example Bill earns more commission than does Sandra. But since Andrew earns more commission than does Lisa, it follows that Bill earns more commission than does Lisa. Any of the following, if introduced into the argument as an additional premise, makes the argument above logically correct EXCEPT: A. B. C. D. E. Andrew earns more commission than Bill Sandra earns more commission than Lisa Sandra earns more commission than Andrew Sandra and Andrew earn the same amount of commission Bill and Andrew earn the same amount of commission

A B and B C , then, A C .

The question asks for an additional premise that does NOT make the argument logically correct. Adding A to the information given in the passage leaves open the possibility that, in order of commission, the people rank: Andrew, Lisa, Bill, Sandra. Because this order is contrary to the conclusion of the argument, A leaves open the possibility that the conclusion of the argument is false; it is thus the best answer. By contrast, any of other choices, when added to the information that the commission Bill earns is greater than that of Sandra and that the commission Andrew earns is greater than that of Lisa, makes the conclusion-that Bill earns more commission than Lisa-follow logically.

86

5. Two Traps Opposites and scope traps are the two most commonly used traps that the test writers use to fool the unskilled test takers. Opposites Trap Here are examples: 1. "Which of the following mostly weakens the above argument?" The test writers present answer that actually strengthens the argument. The earlier the answer appears (say choice A), the more likely it is that you will pick up this answer. 2. The question asks for an assumption for an argument but presents an answer choice that summarizes the argument. Again, the earlier the answer appears (say choice A), the more likely it is that you will pick up this answer. 3. The question asks for an answer that can not strengthen/weaken an argument but provides a choice that can be. For example, Which of the following can not be a possible explanation? The Scope Trap Be alert to scope words that are too narrow or broad. These extreme words are often included in a stuff choice. The answer choice that uses moderate tone (such as usually, sometimes or generally) is generally safe and therefore the correct answer. Extreme Quantifiers

all must

except always

only no

never everywhere

Moderate Quantifiers

probably most

generally many

likely sometimes

some may

87

6. Five Answer Choices On the test day, each of the five choices will follow into one of the categories below. (1) Correct answer choice. The best answer is the correct answer. As we said at the beginning of this chapter, it is not necessarily the truth. Be alert to what the question asks. (2) Opposite answer choice. The opposite answer is exactly opposite to the correct answer. The test writer likes it very much because it takes minutes to create but fool most test-takers. (3) Biased answer choice. The biased choices are also the common trick. Many unskilled test-takers often pick up this acceptable one. The earlier the choice appears, the more students are fooled. (4) Irrelevant answer choice. This choice talks about something that is not related to the passage or the questions. It is easy to eliminate such choice. (5) Biased or Irrelevant answer choice. The fifth answer can be either biased or irrelevant. Example The average net profit margin for the U.S. companies as a whole is 4.5 percent, but companies located in Washing will achieve an average net profit margin of 7.8 percent, and those located in Utah, 3.2 percent. If a firm in Utah were to move to Washing, therefore its net profit margin would be expected to achieve higher profit margin than would be the case if it remained in Utah. Which of the following, if true, would most seriously weaken the conclusion drawn in the passage? A. Consultants in management consulting companies do believe that moving to Washington will significantly increase the average net profit margin. B. The governor of Utah has falsely alleged that statistics for his state are inaccurate. C. The higher net profit margin ascribed to Washingtons current companies is attributable mostly to the small software companies that contributed to over 80 percent of profits generated by all Washingtons companies. D. Thirty percent of all Utahs companies can expect to achieve 7.8 percent of net profit margin. E. Washingtons companies pay tax well below the national average for the United States. Choice A actually strengthens the conclusion.
88

For choice B, because the governor's allegation is false, it cannot affect the conclusion. Choice C suggests that a significant proportion of Washington companies profits are generated by small software companies. Since Utahs companies do not necessarily do business in software industry, this choice presents a reason to doubt that non-software companies will achieve an increased margin. Therefore, this choice is the best answer. Choice D fails to weaken the conclusion because it is consistent with the information given and the conclusion about net profit margin. By suggesting that Washingtons companies pay tax well below the national average for the United States, choice E supports the conclusion.

89

Section 2: Six Types of Argument


There are major two categories of argument, deductive and inductive argument. An argument is deductive if its conclusion necessarily follows from its premise. The most commonly used deductive arguments in GMAT are Mathematical Reasoning and If-Then Theory. Unlike that of the deductive argument, the conclusion of an inductive argument can be highly likely or highly unlikely depending on how convincing the premises are. Typical inductive arguments include Generation, Analogy, and Casual Reasoning. In the following passages, we will introduce them one by one. 1. Deductive Argument A. Statistical Reasoning To reason statistically is to make a conclusion by conducting an arithmetical calculating. For instance, if x = 1and y = 3, then x < y. Of course, this reasoning has its own premise. But this premise is basically indubitable: 1 < 3. In evaluating an argument that contains mathematical calculation, you should try to identify the underlying comparison between two subjects. There are three major concepts that would help you evaluate the validity of a particular argument: comparative value, ratio, and statistical sample.

Concept 1 Comparability: comparative and absolute value There is a big difference between comparative value and absolute value. The former is a ratio, while the later is a value. Comparison of absolute value:

A1 B1

A1 B1 Atotal Btotal Comparison of comparative value:


The following reasoning is ridiculous because it mistakes absolute value as comparative. During the SARS days, about 23,500 doctors who had treated SARS sufferers died and about 23,670 doctors who had not engaged in treatment for SARS sufferers died. On the basis of those figures, it can be concluded that it was not much more dangerous to participate in SARS treatment during the SARS day than it was not to participate in SARS treatment. Which of the following would reveal most clearly the absurdity of the conclusion drawn above? A. Counting deaths among doctors who had participated in SARS treatment in addition to deaths among doctors who had not participated in SARS treatment

90

B.

Expressing the difference between the numbers of deaths among doctors who had treated SARS sufferers and doctors who had not treated SARS suffers as a percentage of the total number of deaths

C. D. E.

Separating deaths caused by accidents during the treatment to SARS suffers from deaths caused by infect of SARS suffers. Comparing death rates per thousand members of each group rather than comparing total numbers of deaths Comparing deaths caused by accidents in the United States to deaths caused by infect in treating SARS suffers.

Concluding from the similar numbers of deaths in two groups that the relative danger of death was similar for both groups is absurd if, as here, one group was far smaller. D exposes this absurdity by pointing out the need to compare death rates of the two groups, which would reveal the higher death rate for the smaller group. Therefore, D is the best answer. Since the conclusion acknowledges the difference between the number of deaths of doctors who treated SARS suffers and doctors who had not treated the SARS suffers, expressing this difference as a percentage, as suggested by B, is beside the point. A is inappropriate because it simply adds a third group to the two being compared. Because cause of death in not at issue, C and E are irrelevant. Concept 2 Ratio This kind of reasoning is similar to the McKinsey's 20-80% rule (20% of our clients generates 80% of total profits). Let's look as an example to see how this rule can help us solve a reasoning problem. A proposed ordinance requires the installation in new taxis of alarm system automatically triggered by the presence of a highjack. However, a taxi driver argued that because more than ninety percent of highjack is alarmed by taxi drive, alarm system would only marginally decrease accidents caused by highjack. Which of the following, if true, would most seriously weaken the taxis drivers argument? A. B. C. D. E. most taxi drivers have no formal training in how to alarm a highjack. Since new taxis are only a tiny percentage of available taxi in the city, the new ordinance would be extremely narrow in scope. The installation of radar in new taxis costs significantly less than the installation of alarm system. In the city where the ordinance was proposed, the average time required by the police department to respond to a highjack was less than the national average. The largest proportion of damage that results from highjack is caused by highjack that starts when taxi drivers fails to alarm.

91

The taxi driver reasons from evidence about most highjack to a conclusion about the effectiveness of alarm system in preventing damage. But this reasoning is faulty because of the possibility that most of the damage results from the minority of highjack excluded from the taxi driver's evidence. That possibility is realized if E is true. Thus, E is the best answer. Because the taxi driver's argument concerns neither the cost of installing alarm systems not a comparison with police department performance in other locations, C and D are irrelevant. The evidence the taxi driver cites suggests that formal training is not needed in order to alarm highjack. So A is not the correct answer. B supports the taxi driver's view that requiring alarm system would have a limited effect. Concept 3 The whole-and-individual The general statistical data does not necessarily apply to an individual. For instance, as a whole, United States is wealthy than China. But we can not say every U.S. resident is wealthier than that in China. To better understand how the whole concept does not apply to an individual, let's look at the following example. Surveys show that in the next 3 years, the population of old age will increase by 20 percent. In order to take this advantage, the department store should replace 20 percent of its goods for young age consumers with goods for old age consumers. Of the following, the best criticism of the conclusion that inducing department store owner to replace goods is that the conclusion is based on A. the fact that the old-age population will increase by less than 20 percent in the town where the department store is located. B. past patterns of consuming and may not carry over to the future C. the assumption that each old consumer is loyal to a single department store at any one time D. the assumption that each department store sells goods for both young and old people E. figures for the retail industry as a whole and may not hold for a particular department store Here, "the population of old age will increase by 20 percent" is a statistical data. But the conclusion is based on figures for the retail industry as a whole and may not hold for a particular department store as demonstrated by the choice of E.

92

B. If-Then Theory If the statement "if A, then B" is true, then its contra-positive "if not B, then not A" is also true. We can diagram this reasoning by followings diagram. If A Example In 2003 an airline in United State lost more than half, on average, of the foreign passengers they had previously served each year. Researchers have alleged that this extreme drop resulted from a rise in price of tickets for international lines from $60 to $90 per 1,000 miles. Which of the following, if feasible, offers the best prospects for alleviating the problem of the drop in passengers as the researchers assessed it? A. Cooperating with other airlines to provide more international lines. B. Allowing foreign passengers to pay the same as the previous international line C. Reemphasizing the goals and mission of the airline as serving both domestic passengers and foreign passengers D. Increasing the financial resources of the airline by raising the ticket price for domestic passengers E. Offering superior VIP service for foreign passengers. The researchers attribute the drop in passengers of foreign passengers to an increased price in ticket. If researchers are correct, reducing these prices should halt the drop in passenger. B offers a plan for reducing these prices and so is the best answer. The reasoning of the stimulus can be expressed as: A (a rise in price of ticket for foreign passengers)=>B (extreme drop of passengers ) Then ~B (drop of passengers did not continue)=>~A (price decreases for foreign passengers) None of C, D and E offers a plan that would reduce the prices taken to be responsible for the drop in passengers. Nor does A offer such a plan: because the problem to be addressed is a drop in foreign passengers, providing more international lines, as A suggests, would offer no prospect of alleviating the problem.

B , then ~ B ~ A .

93

2. Generalization Generalization is an argument that draws a conclusion about a population based on a sample. If the sample is too small or unrepresentative, the conclusion can be fallacious. If the sample is representative or additional sample is added, then the conclusion can be solid. Example Toughened recruiting criterions have not been the primary cause of the present enrollment drop in a certain business school. The drop of enrollment is primarily caused by the fact that in recent years Career Office have not experienced any improvements in assisting graduates to seek jobs. Which of the following, if true, would most support the claims above? A. Many graduates already in the job market would not have been admitted under the new recruiting standards. B. Today more students are entering the school with a higher credential than in the past. C. Some students have cited higher standards for recruiting as a reason for the current enrollment drop. D. Many applicants have cited low quality career help as reasons for not applying to the school. E. Many prospective students have cited the new recruiting standards as a reason for not entering the school. The passage rejects one explanation of the enrollment drop of students-that it results from toughened recruiting standards-and advances an alternative-that it results from deficiencies in career assisting. D provides corroborative evidence for the latter explanation by suggesting that, for applicants, poor career service were reasons for their decision of not applying the school. Therefore, D is the best answer. Choices A, C and E provide evidence that tends to implicate new recruiting standards in the enrollment drop, and thus support the explanation that the passage rejects. B describes what may be a result of the new recruiting standards, but it provides no evidence favoring one explanation of the enrollment drop over the other.

94

3. Analogy Analogy is an argument that two things are similar in this way because they are similar in that way. The greater the similarity between the two things being compared, the stronger the argument will be. Let's look at an example. Example Contrary to the argument made by its opponents, the down-sizing plans are justified. Opponents should remember that the Centralization pulled this company out of great economic troubles even though some of its programs were later found to be unconstitutional. The opponents could effectively defend their position against the author's strategy by pointing out that A. B. C. D. E. the expertise of those opposing the plan is outstanding the lack of justification for the new down-sizing plan does not imply that those who drew it up were either inept or immoral the practical application of the new down-sizing plan will not entail indiscriminate employees cuts economic troubles present at the time of the Centralization were equal in severity to those that have led to the present plan the fact that certain flawed programs or plans have improved the economy does not prove that every such program can do so To attack the author's conclusion, the opponents could point out that even if new down-sizing plan and Centralization are similar in some respects, the other respects of these two programs will not be necessarily the same.

95

4. Causal Reasoning Confusing Correlation with Causation is the most common fallacy associated with causal reasoning. It is clearly questionable to conclude that one thing causes another simply because one happens before the other. It may be only coincidental that they occurred together, or something else may have caused them to occur together. However, if additional evidence is provided, the conclusion can be compelling. Example Our work proves to be very successful. In the past three years, each of our five clients has experienced the fastest growth of sales in their history. Therefore, if your company wants to increase sales, do not hesitate to call Sigma & Max, since we are the solution. Which of the following, if true, most seriously jeopardizes the validity of the argument by the speaker above? A. Most of the consultants at Sigma & Max hold MBA degrees. B. Even without the help of Sigma & Max, the five clients of Sigma & Max will achieve the same growth rate in sale. C. Sigma & Max is one of the five leading management consulting companies. D. Sigma & Max uses an updated accounting approach to help companies to cut cost. E. All of the five clients of Sigma & Max are doing business in financial industry. As B states, without the help of Sigma & Max, the five clients of Sigma & Max will achieve the same growth rate in sale. Therefore, B is the best answer. Choices C and D exactly strengthen the argument by the speaker. The first and last choices are irrelevant to the evaluation of the argument above.

96

5. Finding Assumption While the above four types of logic help you evaluate the effectiveness of an argument in logical thinking, this method assists you in weakening or strengthening an argument by attacking its assumption. As I said at the beginning of this chapter, a typical inductive argument is formulated as: Premises + Assumptions = Conclusion The best way to weaken an argument is to attack its assumption. Let's look at an example: Milk is a perfect food because it provides protein and calcium. The assumption underlying for this reasoning is: Any food that provides protein and calcium is a perfect food. If you can name a food which provides protein and calcium, yet is not a perfect food, then you automatically weaken this argument.

Example #1 If highways were restricted to cars and only those truck with capacity of less than 8 tons, most the truck traffic would be forced to run outside highway. Such a reduction in the amount of truck traffic would reduce the risk of collision in highway. The conclusion draw in the first sentence depends on which of the following assumptions?
97

A. The roads outside highway would be as convenient as highway for most drivers of truck. B. Most roads outside highways are not ready to handle truck traffic. C. Most trucks that are currently running in highway have a capacity of more than 8 tons. D. Cars are at greater risk of becoming involved in collisions than are trucks. E. A reduction in the risk of collision would eventually lead to increases in car traffic. The first sentence concludes that prohibiting trucks with capacity of more than 8 tons from highway would force most trucks away from highways. This conclusion cannot be true unless it is true that, as C says, most trucks that use highways have capacity of more than 8 tons. Therefore, the first sentence's conclusion assumes this choice, which is thus the best answer. The conclusion need not assume that roads outside highways are convenient for trucks (A), since the restrictions would give trucks that have a capacity of more than 8 tons no choice. The conclusion concerns only how the restriction would affect the volume of truck traffic, so B, D and E, which deal with cars and with risk of collision, need not be assumed. Example #2 2. Which of the following, if true, would most strengthen the conclusion drawn in the second sentence? A. Cars with a capacity of more than 8 tons are already excluded outside highways. B. Highways are experiencing overcrowded traffic primarily because of sharp increases in car traffic. C. Many drivers of trucks would rather buy truck with a capacity of less than 8 tons than be excluded from highways. D. The number of collisions that occur near highways has decreased in recent years. E. Trucks that have a capacity of more than 8 tons cause a disproportionately large number of collisions in highways. The second sentence concludes that the reduction described in the first sentence would reduce the risk of collisions in highways. According to E, such a reduction would remove precisely the kind of truck that causes a disproportionate number of collisions. Thus, E is the best answer. Because A does not address the question of whether reducing car traffic would reduce the risk of collisions, it is inappropriate. B and C concern the question of whether or not the proposed restrictions would reduce highway traffic, but not the question of whether any resulting reductions would reduce the risk of collisions. That the number of collisions has recently decreased is irrelevant to whether the proposed reduction would further reduce collisions, so D is inappropriate.

98

6. Business Thinking The test-maker always says that GMAT does not test specific knowledge in business. But as a matter of fact, over one-thirds of GMAT questions are in business subject. In critical reasoning, the percentage is as high as 50%. If you do not possess any business background or knowledge, you will find many questions difficult to answer. Be sensitive to basic business knowledge will definitely help you evaluate an argument. Let's look at an example: Example A life insurance company allows people to prepay their endowment insurance at current rates. The policyholder then pays the premium every year. People should participate in the program as a means of decreasing the cost for their living after retirement. Which of the following, if true, is the most appropriate reason for people NOT to participate in the program? A. Peoples are unsure about which insurance company they will choose after retirement. B. The amount of money accumulated by putting the prepayment funds in an interest-bearing account today will be greater than the total cost of insurance when they retire. C. The annual cost of premium is expected to increase at a faster rate than the annual increase in the cost of living. D. Some of the insurance companies are contemplating large increases in premium next year. E. The prepayment plan would not cover the cost of hospitalization. The passage recommends that people participate in a premium prepayment program as a means of decreasing the living cost after they retire. If B is true, placing the funds in an interest bearing account would be more cost-effective than participating in the prepayment program. Therefore, B would be a reason for NOT participating and is the best answer. A is not clearly relevant to deciding whether to participate since the program applies to whatever insurance companies they choose. C and D, by stating that premium will increase, provide support for participating in the program. E is not clearly relevant to deciding whether to participate, since the expenses mentioned fall outside the scope of the program.

99

Section 3: Eight Types of Question


There are generally eight types of question you expect to encounter on your test day. Among them, Boldface Question is the new one in CAT. You should be familiar with the following question types. 1. Inference Question Inference Question, also known as Conclusion Question, asks you to draw a conclusion based on the stimulus. The following are the typical Inference Questions: Based on the statements above, which of the following can be properly drawn? Which of the following can be implied in the passage above? If the statements above are true, which of the following must also be true? Which of the following inferences is best supported by the statements above? How to solve Inference Question: 1. Read the statement and look for a comparison. The stimulus always compares two things, events, or time sequences. 2. Try to figure out what causes the difference before you look at the answer choices. 3. Use POE to eliminate those choices with extreme quantifiers, such as all, never, must. Example The price of purchasing a car in Country Q is 120 percent less than the price of purchasing a car in Country Y. Even after transportation fees and tariff charges are added, it is still cheaper for a buyer to import car from Country Q to Country Y than to buy car in Country Y. The statements above, if true, best support which of the following assertions? A. Gasoline prices in Country Q are 120 percent below those in Country Y. B. Importing cars from Country Q to Country Y will eliminate 120 percent of the sales of cars in Country Y. C. The tariff on a car imported from Country Q to Country Y is less than 120 percent of the price of a car in Country Y. D. The fee for transporting a car from Country Q to Country Y is more than 120 percent of the price of a car in Country Q. E. It takes 120 percent less time to transport a car in Country Q than it does in Country Y. If the tariff on importing cars from Country Q to Country Y were as high as 120 percent or more of the price of purchasing a car in Y, then, contrary to what the passage says, the price of importing
100

cars from Q to Y would be equal to or more than the price of a car in Y. thus, the tariff cannot be that high, and C is the best answer. A and E give possible partial explanations for the price difference, but neither is supported by the passage because the price advantage in Q might be attributable to other factors. B and D are both consistent with the information in the passage, but the passage provides no evidence to support them.

101

2. Assumption Question The Assumption Question requires you to identify the assumption in an argument. An assumption is a premise that a conclusion depends on but leaves it unstated. Typical Assumption Question looks like this:

The official's conclusion logically depends on which of the following assumptions? The argument above assumes that.

How to solve Assumption Question 1. Read the argument and find its premise and conclusion. 2. Consider what is needed to make the conclusion compelling. 3. Review the answer choices and identify the gap that bridges the premise and conclusion. Example To protect our business secrets, our company could prohibit high-level executives from accepting similar positions in our competitors for three years after such executives leave our company. One such manager concluded, however, that such a prohibition would be unfortunate because it would prevent high-level executives from earning a livelihood for three years. The executive's conclusion logically depends on which of the following assumptions? A. Laws should not restrict the behavior of former company executives. B. Company executives are typically people who have previously been high-level executives from its competitors. C. Low-level company executives do not often seek similar position when they leave company. D. High-level company executives who leave company are capable of earning a livelihood only as lobbyists. E. High-level company executives who leave company are currently permitted to act as lobbyists for only three years. The executive argues that prohibiting high-level company executives from accepting similar positions at its competitors for three years would prevent the executives from earning a livelihood for that period. The reasoning tacitly excludes the possibility of such executives earning a living through work other than in the same industry. Therefore, D, which expresses this tacit assumption, is the best answer. The executive's argument does not depend on the assumption in A, since the argument would not be invalidated if some restrictions on the behavior of company executives were desirable. The executive's argument does not depend on the assumption in B, since the argument would not be invalidated if executives were not typically former high-level company executives of its competitors. The executive's argument does not depend on the assumption in C,

102

since the argument would not be invalidated if former low-level company executives did often become lobbyists. The executive's argument does not depend on the assumption in E, since the argument would not be invalidated if former high-level company executives could act as high-level executive in its competitors indefinitely.

103

3. Strengthen Question Assumptions connect premises to conclusions. You may strengthen an argument by strengthening the assumptions. Here are some examples of Strengthen question stems: Strengthening:

Which of the following statements, if true, would most significantly strengthen the conclusion drawn in the passage? Which of the following, if true, would strengthen the argument presented above?

How to solve Strengthen Questions 1. Read the argument and find its premise and conclusion. 2. Look for assumptions. 3. Read the questions. Use POE until you there is only one choice. Example The average net profit margin for the U.S. companies as a whole is 4.5 percents, but companies located in Washing will achieve an average net profit margin of 7.8 percents, and those located in Utah, 3.2 percents. If a firm in Utah were to move to Washing, therefore its net profit margin would be expected to achieve higher profit margin than would be the case if it remained in Utah. Which of the following statements, if true, would most significantly strengthen the conclusion drawn in the passage? A. As company density increases in Washington, margin figures for that state are likely to be revised downward. B. Market demand factors tending to favor profit margin are abundant in Washington and less numerous in Utah. C. Twenty-five percent of all Utahs companies that move to Washington achieve margin of more than 7 percents. D. Over the last decade, net profit margin has risen at a higher rate for Utahs companies than for Washingtons companies. E. Studies show that the average net profit margin for Utahs companies that move permanently to Washington is roughly equal to that of Washingtons companies that remain in Washington.

104

If B is true, the greater abundance of margin-promoting market demand factors it mentions is probably at least partly responsible for the higher margin in Washington. Therefore, B is the best answer.

105

4. Weaken Question Assumptions connect premises to conclusions. You may weaken an argument by weakening the assumptions. Here are some examples of Weaken question stems: Weakening:

Which of the following statements, if true, would most significantly weaken the conclusion drawn in the passage? Which of the following, if true, would weaken the argument presented above?

How to solve Weaken Question 1. Read the argument and find its premise and conclusion. 2. Look for assumptions. 3. Read the questions. Use POE until you there is only one choice. Example The average net profit margin for the U.S. companies as a whole is 4.5 percent, but companies located in Washing will achieve an average net profit margin of 7.8 percent, and those located in Utah, 3.2 percent. If a firm in Utah were to move to Washing, therefore its net profit margin would be expected to achieve higher profit margin than would be the case if it remained in Utah. Which of the following, if true, would most seriously weaken the conclusion drawn in the passage? A. Consultants in management consulting companies do believe that moving to Washington will significantly increase the average net profit margin. B. The governor of Utah has falsely alleged that statistics for his state are inaccurate. C. The higher net profit margin ascribed to Washingtons current companies is attributable mostly to the small software companies that contributed to over 80 percent of profits generated by all Washingtons companies. D. Thirty percent of all Utahs companies can expect to achieve 7.8 percent of net profit margin. E. Washingtons companies pay tax well below the national average for the United States. Choice A actually strengthens the conclusion. For choice B, because the governor's allegation is false, it cannot affect the conclusion.

106

Choice C suggests that a significant proportion of Washington companies profits are generated by small software companies. Since Utahs companies do not necessarily do business in software industry, this choice presents a reason to doubt that non-software companies will achieve an increased margin. Therefore, this choice is the best answer. Choice D fails to weaken the conclusion because it is consistent with the information given and the conclusion about net profit margin. By suggesting that Washingtons companies pay tax well below the national average for the United States, choice E supports the conclusion.

107

5. Paradox Question The Paradox Question presents you with a seeming paradox situation in an argument, and then asks you to seek an explanation on how that discrepancy can exist. Typical ways in which these questions are asked include:

Which of the following statements, if true, would best explain the paradox described above? Which of the following can explain the apparent contradiction above?

How to solve paradox question 1. Read the argument and identify the apparent contradiction. 2. Look for the sequential difference or spatial difference since they may be the reason that causes the seeming paradox. 3. If you are unable to pick up the correct answer by step 2, pay attention to the subjects in the two contradictive situation. Are the two subjects the same? If not, the difference could be the reason for paradox. Example In 1992, 5 percent of every dollar paid in tax went to support the unemployed citizens. In 1998, 8 percent of every dollar paid in tax went to such funds, although that unemployment rate has decreased in 1998 than in 1992. Each of the following, if true, could explain the simultaneous increase in percent of every dollar paid in tax to support the unemployed citizens and decrease in the number of unemployment rate EXCEPT: A. On average, each unemployed citizen received more money in 1998 than 1992. B. On average, people paid less tax in 1998 than in 1992. C. The individuals had paid more tax than did enterprises during this period. D. Income before tax has significantly decreased since 1992. E. The number of tax evaders rose sharply between 1992 and 1998. Choice A suggests that the total amount of dollars used to support unemployment has increase, therefore explain the paradox. Choice B, D, and E all suggests that the amount of tax collected decreased, thus percent of every dollar that went to support the unemployment increases. Only choice C does not explain such paradox, therefore is the correct answer.

108

6. Reasoning Question Reasoning questions ask you to describe how the line of reasoning of an argument. Here are some examples:

The author's point is made by. A attacks Bs position by. How does the author make his point?

How to solve Reasoning Question 1. Read the argument and identify its premise and conclusion. 2. Identify how the premises and/or evidences lead to the conclusion. 3. Pick up the correct answer based on your awareness of typical reasoning methods. The most commonly used reasoning ways are Analogy, Generalization, Causal Reasoning and Attacking Assumption. Example Contrary to the argument made by its opponents, the down-sizing plans are justified. Opponents should remember that the Centralization pulled this company out of great economic troubles even though some of its programs were later found to be unconstitutional. The author's method of attacking the opponents argument is to A. B. C. D. E. attack the character of the opponents rather than their claim imply an analogy between the current down-sizing plan and a previous Centralization program point out that the opponents' claims imply a dilemma show that the opponents' reasoning leads to an absurd conclusion show that the Centralization also called for down size the employee pool

The argument of the stimulus goes like this: the new down-sizing plan and Centralization are similar to each other because both are improper. Since the Centralization pulled this company out of great economic troubles, therefore, the new plans of down size are justified. Obviously, the speaker used analogy to make a conclusion.

109

7. Complete Question As its name suggests, this type of question asks you to complete a reasoning that contains a blank. This type is seldom in GMAT critical reasoning. However, if you do encounter it on the test day, this passage will help you quickly identify the right answer choice.

The blank needed to complete is always a part of the reasoning presented in the stimulus. That is to say, one of the following five choices, will bridge the premise and conclusion. So, what you need to do is to find a statement or fact that would make the above argument complete, reasonable and persuasive. How to approach Complete Question 1. Look for gaps between the premises and the conclusion. Ask yourself why the conclusion is true. Before you progress to the answer choices, try to get feel of what statement is necessary to fill that gap between the premises. 2. Eliminate the choices that can't make the argument reasonable.

Example Which of the following best completes the passage below? In a survey of law school applicants, two-fifths admitted to create at least one reference letter by themselves. However, the survey may underestimate the proportion of law school applicants who create reference letter by themselves, because____. A. some applicants who create reference letter by themselves taking the survey might have claimed on the survey not to create reference letter by themselves B. some law school applicants who create reference letter by themselves taking the survey might have claimed on the survey to create reference letter by themselves C. some law school applicants who claimed on the survey to create at least one reference letter may create more than one reference letter D. some law school applicants who claimed on the survey to create reference letter by themselves may have been answering exactly E. some applicants who are not applying for law school probably create at least one reference letter A is the best answer. If law school applicants who in fact create reference letter by themselves claimed not to create reference letter by themselves, the survey results would show a smaller proportion of law school applicants who create reference letter by themselves than actually exists.

110

Therefore, this choice is the best answer. B is inappropriate because generally law school applicants who do not create reference letter by themselves claimed to create reference letter by themselves could contribute to the overestimation, but not to the underestimation, of applicants who create reference letter by themselves. D is inappropriate because applicants who admitted that they create reference letter by themselves would not contribute to an underestimation of the proportion of applicants who create reference letter by themselves. C and E are inappropriate because the argument is concerned neither with the number of reference letters created by applicants nor with the applicants to non-law school.

111

8. Boldface Question In this type, one or more sentences in the stimulus are in boldface. The question following requires you to identify the logical relationship between the boldfaced sentences, or how it relates to a particular position (the author agree or disagree). Boldface question is totally new for computer-based test, but is easy to tackle. All you have to do is to understand the argument: identify the conclusion, evidence, and the reasoning from evidence to conclusion. Sometime, you are required to critique the validity of the argument. How to approach Boldface Question 1. Identify the conclusion. Ask yourself what the conclusion is, what the author trying to prove, or what the authors main point is. 2. 3. Look for the evidence that the author uses to support or argue against a position. Search for argument indicator to determine the relationship between evidence and conclusion. Conclusion Indicators so consequently conclude that Premise Indicators because if on the basis of since assume the reason is that for suppose may be derived from as evidence in that thus accordingly follows that therefore hence means that as a result imply infer that

Counter-evidence Indicators actually even though however despite nonetheless In spite of admittedly nevertheless do except although may

112

4. Before you pick up the answer choices, make sure you did NOT mistake counter-evidence as evidence. Let's look at a relatively easy question. Example #1 To be eligible for a U.S. student visa, an applicant must have a valid passport, and have been accepted by a U.S. institution. Peter, a French applicant, has a valid passport and been admitted to Western California State College, so he must be issued a U.S. student visa. The two portions in boldface play which of the following roles? A. The first is the part of evidence in support of this argument; the second is the conclusion that could not be drawn from all evidence that the argument contains. B. The first is the first-evidence that supports this argument; the second is the main point that must be drawn from all evidence that the argument includes. C. The first is the one fact of two that argument includes; the second is the conclusion that could be drawn from this passage. D. The first is the background that is necessary for this argument; the second is the conclusion that is not drawn only from the first. E. The first is the cause that the argument includes; the second is the effect that can be drawn only from this cause. In this argument, the author first introduced the requirement to apply for a U.S. student visa. The author then concluded that since Peter meets the requirement, he will necessarily get the visa. It is obviously that this conclusion can not be properly drawn because application can be denied even that the applicant meets the basic requirement. Therefore, A is the best answer. It seems easy. Now take a high-level question. Example #2 Something must be done to stop spam. In early days, people seldom received unsolicited email advertisement; but now that numerous bulk email software and email address finders are developed to collect email address all around the world. Advertisers use email addresses to market their products and even sell such email lists to other advertisers. As a result, almost everyone ever get junk email, and sometime several and even tens of annoying emails a day. So, relevant anti-spam regulations should be framed to stop unsolicited advertising. The two portions in boldface play which of the following roles?

113

A. Background that the argument depends on and conclusion that can be drawn from the argument. B. Part of evidence that the argument includes, and inference that can be drawn from this passage. C. Pre-evidence that the argument depends on and part of evidence that supports the conclusion. D. Background that argument depends on and part of evidence that supports the conclusion. E. Pre-evidence that argument includes and a method that helps to supports that conclusion. The first portion in boldface introduced a previous situation, as compared to current situation. The author then made the conclusion in the last sentence, or the second portion in boldface. Therefore, choice B is the best answer.

The End of GMAT Verbal Study Guide

114

Vous aimerez peut-être aussi